Download as pdf or txt
Download as pdf or txt
You are on page 1of 53

Chapter 

22  Quantitative techniques in business 150

MULTIPLE CHOICE

Probability
1. A decision maker is operating in an environment in which all the facts surrounding a
decision are known exactly, and each alternative is associated with only one positive
outcome. The environment is known as
A. Certainty. C. Uncertainty
B. Risk. D. Conflict.

1. A
? The environment in which all the facts surrounding a decision are known exactly.
 When all the facts surrounding a decision are known exactly, and each alternative is
associated with only one positive outcome, then such decision environment is
operating under the condition of certainty. Hence, letter-choice “a” is correct.
Choice-letter “b” is incorrect because it refers to the concept of conditional
probability where there is uncertainty as to the occurrence of non-occurrence of a
future event. As such, there is always a risk that such environment may or may not
occur. Choice-letter “c” is incorrect because uncertainty is the same as risk, and
therefore there is no exactness (or certainty) as to the occurrence or non-occurrence
of an event. Choice-letter “d” is incorrect because conflict does not relate to the
chance that a future event may occur or not.

2. A company uses two major material inputs in its production. To prepare its
manufacturing operations budget, the company has to project the cost changes of
these material inputs. The cost changes are independent of one another. The
purchasing department provides the following probabilities associated with projected
cost changes:
Cost Change Material 1 Material 2
3% increase .3 .5
5% increase .5 .4
10% increase .2 .1
The probability of 3% increase in the cost of both Material 1 and Material 2 is
A. 15% C. 80%
B. 40% D. 20%

2. A
? The probability of a 3% increase in the cost of both Material A and Material B.
 The probability that material A would change given a 3% increase in cost is 30%,
while the probability that material B would change given a 3% increase in cost is
50%. Therefore, the joint probability the both material A and B would change given a
3% increase in cost would be 15% (i.e., 30% x 50%). Hence, choice-letter “a” is
correct.
Choice-letter “b” is incorrect because 40% is the simple average of 30% and
50%, and not the joint probability required by the problem. Choice-letter “c” is
incorrect because it gets the sum of 30% and 50%. Choice-letter “d” is incorrect
because it gets the difference of 50% and 30%, not the joint probability.
Chapter 22  Quantitative techniques in business 151

Questions 3 and 4 are based on the following information. The probabilities shown in
the table represent the estimate of sales for a new product.
Sales (Units) Probability
0 – 200 15%
201 – 400 45
401 – 600 25
601 – 800 15

3. What is the probability of selling between 201 and 600 units of the product?
A. 0% C. 70%
B. 11.25% D. 25%

3. C
? The probability of selling between 201 and 600 units of the product.
 The total probability of selling between 201 to 600 units is 70% (i.e., 45% + 25%).
This covers the range of 201 to 400 units having 45% probability and 401 to 600 units
having 25% probability.

4. What is the best estimate of the expected sales of the new product?
A. 480 C. 400
B. 380 D. 800

4. B
? The best estimate of the expected sales of the new product.
 The expected sales of the new product is the sum of all its probabilities multiplied by
the future value. The values of the future events (e.g., sales in units) are given in
range (0-2--, 201-400, etc.) The first thing to do is to get the mid-point of the values
in the range before multiplying the same to their probabilities. The expected sales is
380 units computed as follows:
Sales (in units) Mid-point Probability Expected sales
0 – 200 100 15% 15
201 – 400 300 45 135
401 – 600 500 25 125
601 – 800 700 15 105
Total 380

5. Ryerson Company has three sales departments, each contributing the following
percentages of total sales: clothing, 50%; hardware, 30%; and household sundries,
20%. Each department has had the following average annual damaged goods rates:
clothing, 2%; hardware, 5%; and household sundries, 2.5%. A random corporate
audit has found a weekly damaged goods rate of sufficient magnitude to alarm
Ryerson’s management. The probability (rounded) that this rate occurred in the
Clothing Department is:
A. 50% C. 25%
B. 1% D. 33 1/3 %

5. D
? The probability that an error rate occurred in the Clothing Department.
Chapter 22  Quantitative techniques in business 152

 The probability that an error rate occurred in the Clothing Department is based on its
specific joint probability over that of the future events’ total joint probability, as
follows:
Total sales Error rates Joint Probability
Clothing 50% 2,0% 1.0%
Hardware 30% 5.0% 1.5%
Household Sundries 20% 2.5% 0.5%
Total 3.0%
Therefore, the probability that an error would occur in the Clothing Department is
33 1/3 % (i.e., 1% / 3%).

6. A quantitative technique useful in projecting a firm’s sales and profits is


A. Probability distribution theory. C. Learning curves.
B. Gantt chart. D. Queuing theory.

6. A
? A quantitative technique useful in projecting a firm’s sales and profits.
 Choice-letter “a” is correct because probability distribution theory refers to the
technique of estimating the value of future events based on various rates of
occurrences. Choice-letter “b” is incorrect because Gantt Chart is a project
scheduling technique used to monitor project activities in terms of organized activity
sequencing and controlling in the expected time of completing a project. Choice-
letter “c” is incorrect because learning curve is primarily used in estimating labor cost
to be incurred for every doubling of output. Choice-letter “d” is incorrect because
queuing theory relates to the technique of balancing the cost of providing the service
facility to reduce the cost of waiting.

7. Two firms share customers in the same market. Firm A sampled its customer’s
buying habits and found that about 70% of its customers were repeat customers each
week, while 30% went to Firm B. Firm B found that 80% of its customers remained
loyal each week, while 20% switched to Firm A. If this retention and loss of
customers continues for a long period, the percentage of customers Firm A will have
is
A. 70% C. 60%
B. 80% D. 40%

7. D
? The percentage of customers firm A will have if the retention and loss of customers
continue for a long period.
 The total market is 100%. This market is divided between firms A and B. Since 30%
of firm A’s sales go to firm B, then firm B shares 30% of the total transferring market.
And since 20% of firm B’s sales go to firm A, then firm A shares 20% of the total
transferring market. The total transferring market is 50% (i.e., 30% + 20%).
Therefore, the share of firm A in the market is 40% (i.e., 20% / 50%).

Questions 8 and 9 are based on the following information: A computer store sells
four computer models designated as P104, X104, A104 and S104. The store
Chapter 22  Quantitative techniques in business 153

manager has made random number assignments to represent customer choices


based on past sales data. The assignments are shown below:
Model Random Numbers
P104 0-1
X104 2-6
A104 7-8
S104 9

8. The probability that a customer will select model P104 is


A. 10% C. 50%
B. 20% D. 30%

8. B
? The probability the a customer will select model P104.
 The probability that an event may happen refers to its rate of occurrence based on
total possible occurrences which is determined by the number of assigned random
numbers.
Number of Assigned
Model Random Numbers Random Numbers Probability
P104 0-1 2 20%
X104 2-6 5 50
A104 7-8 2 20
S104 9 1 10
10 100%
Therefore, the probability that a customer would select model P104 is 20%.

9. In running a simulation of a computer demand, the following numbers are drawn in


sequence: 2, 8, and 6. The simulation indicates that the third customer will purchase
A. Model P104 C. Model A104
B. Model X104 D. Model S104

9. B
? The model expected to be purchased by a customer based on a simulation results.
 The simulation results to a sequence of 2, 8, and 6. This indicates that the third
customer would probably purchase the model where random number 6 is assigned.
Random number 6 is assigned to model X104, hence, this is the model the third
customer would probably purchase.

10. Probability (risk) analysis is


A. Used only when the sum of probabilities is less than one.
B. Used only for situations involving five or fewer possible outcomes.
C. Incompatible with sensitivity analysis.
D. An extension of sensitivity analysis.

10. D
? A true statement regarding probability or risk analysis.
Chapter 22  Quantitative techniques in business 154

 Choice-letter “d” is correct because probability analysis is an extension of sensitivity


analysis. It deals with determining possible outcomes given the changes in the
variables that affect such outcomes. This relates to the determination of future
events which uses probability techniques. Choice-letter “a” is incorrect because the
total of all conditional probabilities is always 1.0. Choice-letter “b” is incorrect
because probability analysis is used even for situations involving more than five
outcomes. Choice-letter “c” is incorrect because probability analysis is compatible,
not incompatible, with sensitivity analysis.

Statistics
Questions 11 through 13 are based on the following information. Pace Company’s
accountant has gathered a sample of sales volume in units for the past 11 working
days.
Day Sales Volume in Units
1 100
2 150
3 200
4 100
5 100
6 300
7 135
8 180
9 160
10 190
11 310
1,925
11. The mean value for daily sales for this period is
A. 1,925 C. 160
B. 100 D. 175

11. D
? The mean value for daily sales.
 The mean value (or the average value) may be determined using a simple mean (or
simple average) or weighted mean (or weighted average mean). In this problem, the
mean referred to is the simple average mean which is determined by dividing the
total of the sales volume with the number of days. The mean value is 175 units (i.e.,
1,925 / 11).

12. The modal value for daily sales for this period is
A. 300 C. 160
B. 100 D. some amount other than those given.

12. B
? The modal value of the daily sales.
 The modal value is the most occurring observation. Among the observations (i.e.,
sales volume in units) given, the most occurring observation is 100 units; it occurred
for 3 times, and is the most number of occurrences.
Chapter 22  Quantitative techniques in business 155

13. The median value for daily sales for this period is

A. 500 C. 160
B. 100 D. 175

13. C
? The median value of the daily sales.
 The median value is the mid-point of all the observations after arranging from lowest
to highest. In doing so, the arrangement appears as follows:
Observation Sales volume in units
1 100
2 100
3 100
4 135
5 150
6 160 (mid-point)
7 180
8 190
9 200
10 300
11 310
The 6th observation is the mid-point between 1 and 11. And the mid-point is 160,
the value of the 6th observation.

14. A sales supervisor is concerned with what appear to be unusually high travel
expenses of one salesperson. The measure by which the sales supervisor can best
evaluate the difference between the expenses of one salesperson and those of other
salespersons is the
A. Coefficient of correlation. C. Median.
B. Mode. D. Standard deviation.

14. D
? The measure that best evaluates the difference between the expenses of one
salesperson and those of other salespersons.
 Choice-letter “d” is correct because standard deviation measures the range of
expenses of a salesman with the other salesmen. This would permit the manager to
monitor and check unusual travel expenses of a salesperson.
Choice-letters “a”, “b”, and “c” are incorrect answers because they are not useful
in measuring the variation of an event. Choice-letter “a” is incorrect because
coefficient of correlation is a measure to determine the relationship of a dependent
variable with an independent variable. Choice-letters “b” and “c” are incorrect
because mode and median are measures of central tendency and not of variability.

Questions 15 through 17 are based on the following information. The management


at Treasure Corporation is evaluating two new products to undertake during the next
1-year period, digital clocks and calculators. For each product, the Marketing
Chapter 22  Quantitative techniques in business 156

Department has estimated the possible net cash flows and the probability of each
cash flow.

Digital Clocks Calculators


Probability Cash Flow Probability Cash Flow
25% P4,000 10% P 0
50 5,000 50 3,000
25 8,000 40 10,000
Since both products have equal expected values of P5,500, management is still
undecided. James, a financial analyst, has suggested that the calculation of the
standard deviation for each product’s cash flows may aid in the decision. He used the
following data:
____________
S =  (Xo – X)2 Pe
n

If: e=1
Xo = individual cash flows
X = expected value of all cash flows
Pe = probabilities assigned to each cash flow

15. James is using the standard deviation to assist in the decision process because
he is
A. Measuring the size of the constraints.
B. Determining the frequency of an event.
C. Measuring the dispersion of the probability for each event.
D. Measuring the range of each event.

15. D
? The reason for the use of standard deviation.
 Choice-letter “d” is correct because standard deviation is a measure of variability or
dispersion. Standard deviation measures the variability of the range of each event.
Choice-letter “a” is incorrect because measuring the size of the constraint is a
concern of linear programming which deals with the optimization of scarce resources.
Choice-letter “b” is incorrect because determining the frequency of an event would
help in establishing the central tendency (e.g., mean) and not the variability. Choice-
letter “c” is incorrect because standard deviation does not measure the dispersion of
the probability of an event but rather the dispersion from the mean.

16. The standard deviation of the cash flows of the digital clock is
A. P2,250,000.00 C. P750.00
B. P1,500.00 D. P353.55

16. B
? The standard deviation of the cash flows.
 The standard deviation is the square root of the variance (r²). First, let us get the
mean (x bar); then, get the variance; and last, the standard deviation. The standard
deviation of the product digital clock shall be:
(a) X bar = P5,500 (given in the problem)
Chapter 22  Quantitative techniques in business 157

(b) Variance =
X X bar (X – X bar)² Probability (X – X bar)² x Probability
P4,000 P5,500 2,250,000 25% 562,500
5,000 5,500 250,000 50% 125,000
8,000 5,500 6,250,000 25% 1,562,500
2,250,000

Standard deviation = 2,250,000 = 1,500

17. If the standard deviation for the cash flows of the calculators is P3,775, the
management at Treasure would
A. Choose to introduce digital clocks because this product has
the lesser risk.
B. Choose to introduce calculators because this product has a
40% probability for the highest cash flow.
C. Not choose either product.
D. Choose to introduce calculators because the product has the
lesser risk.

17. A
? The management decision given the standard deviations of products digital clocks
and calculators.
 The standard decision of digital clock is P1,500 (please refer to the preceding
question) while the standard deviation of calculator is P3,775. Standard deviation
measures the range of variability (or dispersion, or risk). The higher the standard
deviation, the higher the risk of a given product. Since the standard deviation of
product digital product is lower, it has lesser risk, and is therefore a better alternative.
Take not that the products have the same expected values. Hence, choice-letter “a”
is correct.
Choice-letter “b” is incorrect because the probability for the highest cash flow is
not a measure of risk and expected payoff. Choice-letter “c” is incorrect because
there is no indication given on the problem that the products are unprofitable but
rather the management is simply evaluating which product to choose. Choice-letter
“d” is incorrect because calculator has a higher risk since it has greater standard
deviation.

18. Statistical quality control often involves the use of control charts whose basic purpose
is to
A. Determine when accounting control procedures are not
working.
B. Control labor costs in production operations.
C. Detect performance trends away from normal operations.
D. Monitor internal control application in computerized operations.

18. C
? A basic purpose of using control charts.
 Control charts are used to keep performance within the normal (or standard) range
and monitor performance away from normal operations; choice-letter “c” is correct.
Chapter 22  Quantitative techniques in business 158

Choice-letter “a’ is incorrect because control charts are not substitute, but rather
complementary, to accounting controls. Choice-letter “b” is incorrect because control
charts do not have a direct impact in controlling labor costs in the operations but
rather give managers a report and impression on operations. Choice-letter “d” is
incorrect because control charts do not monitor the internal control operations in a
computerized operations but monitor the actual results and help in determining
performance deviation from normal conditions.

19. A Philippine Company currently has domestic operations only. It is considering an


equal-size investment in either Canada or Britain. The data on expected rate of return
and the risk associated with each of these proposed investments are given below.
Proposed investment Mean Return Std. Deviation
British investment 22% 10%
Canadian investment 28% 15%
The mean return on the company’s current (domestic only) business is 20%, with a
standard deviation of 15%. Using the above data and the correlation coefficients, the
company calculated the following portfolio risk and return (based on a ratio of 50%
Philippine domestic operations and 50% international operations).
Investments Mean Return Std. Deviation
Phil and Britain 21% 3%
Phil and Canada 24% 15%
The company plans to select the optimal combination of countries based on risk and
return for the domestic and international investments taken together. Based on the
above data, which one of the following alternatives provides the best risk adjusted
return to the firm?
A. Undertake the British investment.
B. Undertake the Canadian investment.
C. Do not undertake either investment.
D. Unable to determine based on data given.

19. A
? The alternative that provides the best risk adjusted return to a firm.
 The best risk adjusted return is measured by the coefficient of variation which is the
computed by dividing standard deviation over the mean. The coefficient of variation
of the possible combined operations (i.e., domestic and foreign) shall be:
Investment mix Standard Deviation x Mean Ratio = Coefficient of Variation
Phil. and Britain 3% 21% 14.32%
Phil. and Canada 15% 24% 62.50%
The investment mix that would report the lower coefficient of variation is a less
risky investment and would be the better choice. Therefore, choice-letter “a” is
correct because it has a lower coefficient of variation at 14.32%.

20. A quantitative technique used to discover and evaluate possible cause-and-effect


relationship is
A. Correlation analysis. C. Program evaluation review technique (PERT).
Chapter 22  Quantitative techniques in business 159

B. Linear programming. D. Poisson distribution models. (rpcpa)

20. A
? A quantitative technique used to discover and evaluate possible cause-and–effect
relationship.
 Choice-letter “a” is correct because correlation analysis determines the relations of
independent variable (x) with respect to dependent variable (y). Variable “Y” changes
as independent variable “x” changes. The causal change in “x” has an effect in “y”.
Choice-letter “b”, linear programming, is incorrect because linear programming
deals with analyzing the best use of scarce resources for optimum objective function.
Choice-letter “c”, PERT, is incorrect for it is used in estimating the best possible path
of activities in completing a project.
Choice-letter “d” is incorrect because Poisson distribution model is useful when
the event being studied may happen more than once with random frequency during a
given period.

21. A regression equation


A. Estimates the dependent variables.
B. Encompasses factors outside the relevant range.
C. Is based on objective and constraint functions.
D. Estimates the independent variable.

21. A
? A statement describing regression analysis.
 Choice-letter “a” is correct because regression analysis estimates the dependent
variable in the equation Y = a + bx. The dependent variable is “Y” which is to be
determined based on the constant values of “a” and “b” and the varying value of “x”.
Choice-letter “b” is incorrect because regression equation depicts a straight line, a
linear line, and is applicable within the relevant range. Choice-letter “c” is incorrect
because objective and constraint functions are of linear programming and not
regression equation. Choice-letter “d” is incorrect because regression equation does
not estimate the value of independent variable, but that of dependent variable.

22. The correlation coefficient that indicates the weakest linear association between two
variables is
B. -0.73 C. 0.12
C. -0.11 D. 0.35

22. B
? The coefficient correlation that indicates the weakest linear association between two
variables.
 The coefficient of correlation measures the relationship of the dependent variable
with the independent variable. The correlation may be positive, negative, or zero.
The value of the correlation coefficient should be within the range of “-1.00 to +1.00”.
The nearer it is to the value of 1.00, either positive or negative, means a presence of
stronger correlation between variables “Y” and “x”. The nearer it is to zero means a
presence of weaker correlation between variables “Y” and “x”. Choice-letter “b” is
correct because it has the lowest correlation coefficient.
Chapter 22  Quantitative techniques in business 160

A positive correlation happens when the dependent variable changes in the


same fashion with the independent variable. That is, when the independent variable
increases, the dependent variable likewise increases; and when the independent
variable decreases, the dependent variable also decreases. A negative correlation
happens when the dependent variable changes in inverse relation with the
independent variable such that when the independent variable increases, the
dependent variable decreases, and vice-versa.

23. What coefficient of correlation results from the following data?


X Y
1 10
2 8
3 6
4 4
5 2

A. 0 C. +1
B -1 D. Cannot be determined from the data given.

23. B
? The description of the coefficient of correlation resulting from the data given on the
values of “X” and “Y”.
 The relationship of variables X and Y is negative or inverse. When X value
increases, Y value decreases, and vice-versa. The observation shows a perfect
negative correlation between variables X and Y; choice-letter “b” is correct.

Questions 24 through 27 are based on the following information. In preparing the


annual profit plan for the coming year, A Company wants to determine the cost of
behavior pattern of the maintenance costs. A Company has decided to use linear
regression by employing the equation y = a + bx for maintenance costs. The prior
year’s data regarding maintenance hours and costs, and the results of the regression
analysis are given below.
Average cost per hour P9.00
a 684.65
b 7.2884
Standard error of a 49.515
Standard error of b .12126
Standard error of the estimate 34.469
r2 .99724
Hrs. of Maintenance
activity cost
January 480 P4,200
February 320 3,000
March 400 3,500
April 300 2,820
May 500 4,350
June 310 2,960
July 320 3,030
Chapter 22  Quantitative techniques in business 161

August 520 4,470


September 490 4,260
October 470 4,050
November 350 3,300
December 340 3,160
Sum 4,800 P43,200
Average 400 P3,600

24. In the standard regression equation y=a + bx, the letter b is best described as a(n)
A. Independent variable C. Constant coefficient
B. Dependent variable D. Variable coefficient

24. C
? The description of the variable “b” in the standard regression equation.
 Choice-letter “c” is correct. In the standard regression equation Y = a + bx, “b” is the
constant coefficient of variable “x”. To identify the variables in the equation, “Y” is the
dependent variable, “a” is the intercept or the constant variable, “b” is the variable
coefficient or the slope, and “x” is the independent variable.
Choice-letter “a” is incorrect because independent variable refers to “x”. Choice-
letter “b” is incorrect because dependent variable refers to “Y”. Choice-letter “d” is
incorrect because “b” is a constant, not a variable, coefficient of “x”.

25. The letter x in the standard regression equation is best described as a(n)
A. Independent variable C. Constant coefficient
B. Dependent variable D. Variable coefficient

25. A
? The description of the variable “x” in the standard regression equation.
 Choice-letter “a” is correct because the variable “x” refers to the independent
variable.

26. Based upon the data derived from the regression analysis, 420 maintenance costs
(rounded to the nearest peso) would be budgeted at
A. P3,780 C. P3,790
B. P3,600 D. P3,746

26. D
? The estimated cost of using 420 maintenance hours, using the standard regression
equation.
 The standard regression equation is Y = P684.65 + P7.2884x. The value of “x” is
4420 hours. Applying, the value of Y would be P3,746 [i.e., Y = P684.65 +
P7.2884(420)].

27. The percentage of the total variance that can be explained by the regression
equation is:
A. 99.724% C. 80.982%
B. 69.613% D. 99.862%
Chapter 22  Quantitative techniques in business 162

27. A
? The percentage of the total variance that can be explained by the regression
equation.
 The variance of the standard regression is represented by r², which is already given
with a value of 0.99724 or 99.724%.

28. Regression analysis is superior to other cost behavior analysis techniques because it

A. Examines only one variable.


B. Produces measures of probable error.
C. Proves a cause and effect of relationships.
D. Is not a sampling technique. (rpcpa)

28. B
? The reason why regression analysis is superior to other cost behavior analysis.
 Regressions analysis is a statistical technique use to track down the pattern of
movements and behavior of total costs fixed cost and variable costs. It uses the
straight-line equation Y = a + bx, where:

Y = dependent variable (to be computed)


a = point of intercept or origin (total fixed costs)
b = slope, coefficient of “x”, or constant coefficient (variable cost rate)
x = independent variable (units of measure)
Being a statistical approach, it uses sampling techniques (choice-letter “d” is
incorrect). It examines more than one variable (choice-letter “a” is incorrect). Choice-
letter “c”, about cause and effect relationship, is more of a concept of probability and
not of statistics.

29. B Company is an automobile replacement parts dealer in large metropolitan


community. B Company is preparing its sales forecast for the coming year. Data
regarding both B Company’s and industry sales of replacement parts as well as both
the used and new B Company sales in the community for the last 10 years have
been accumulated. If B Company wants to determine if there is a historical trend in
the growth of its sales as well as the growth of industry sales of replacement parts,
the company would employ
A Simulation technique. C. Statistical sampling.
B. Queuing theory. D. Time series analysis.

29. D
? The technique used to determine the historical trend in the growth of sales as well as
the growth of industry sales of replacement parts of a company.
 Choice-letter “d” is the correct answer. Time series analysis is a regression
technique that depicts the trend relationship of a value in relation to time. Choice-
letter “a” is incorrect because simulation technique is used to predict the outcome of
a value given a change in its parameters. Choice-letter “b” is incorrect because
queuing theory determines the optimum relationship of a savings from providing a
service center with its attendant cost. Choice-letter “c” is incorrect because statistical
Chapter 22  Quantitative techniques in business 163

sampling speaks of the process used in deriving conclusions based on the


representations of items from the entire population.

30. A health insurance company uses a computer application to monitor physician bill
amounts for various surgical procedures. This program allows the company to better
control reimbursement rates. The X-bar chart below is an example of the output from
this application.

XX
XX
Upper limit XXX
XXX X
X XXXX
XX X XX X XXX
Expected Mean X XX X X X XX X XX X
X X X X X XX X
X X X
X
Lower limit
Select the interpretation that best explains the data plotted on the chart.
A. Random variation. C. Normal variation.
B. Abnormal variation. D. Cyclic variation.

30. B
? The interpretation that best explains the data plotted on the chart.
 Choice-letter “b” is correct; because the data on the graph depicts an abnormal
variation as several observations are plotted outside the normal limits. Choice-letter
“a” is incorrect because the observations plotted on the graph are initially clustered
within the normal limits and later observed to have been going out of the normal
limits. This indicates a trend in the observation. The observations are not randomly
scattered on the graph rather informs the reader that things are veering away from
normal expectations. Choice-letter “c” is incorrect since the latest observations are
showing abnormality from standards. Choice-letter “d” is incorrect because the
observations presented on the graph do not show cyclical trend (say, ups and downs
in a cycle), rather of a clustered observation within the normal limits that have
recently exceeded the normal expectations.

Expected value
31. S Company is preparing its 2006 budget and, taking into consideration the recent
pace of economic recovery, has developed several sales forecasts and the estimated
probability associated with each sales forecast. To determine the sales forecast to be
used for 2006 budgeting purposes, which one of the following techniques should S
use?
A. Expected value analysis. C .Monte Carlo simulation.
B. Continuous probability simulation.. D. Sensitivity analysis.

31. A
? The technique for determining a sales forecast given different forecast and their
probabilities.
Chapter 22  Quantitative techniques in business 164

 Choice-letter “a” is correct. Sales forecast represents the expected market


performance of the business in a given future period. This expectation may be based
on various scenarios that may happen in the future. Probabilities are assigned to
various future events and their equivalent values are totaled. The total of the
different equivalent values of future events is the expected value.
Choice-letter “b” is incorrect since simulation is no longer needed because
estimates are already known. Choice-letter “c” is incorrect because the exponential
distribution analysis is the probability of zero occurrence in a defined time period.
Choice-letter “d” is incorrect sensitivity analysis is used to determine the expected
result in a future event given the changes in the variables affecting such event.
Sensitivity and simulation analysis are used to determine the expected future events
without regard to their assigned probability of occurrence. Once the assigned
probabilities are used to adjust the values of conditional future events, the process of
expected value analysis takes into effect.

32. The expected value of perfect information is the


A. Same as the expected profit under certainty.
B. Sum of the conditional profit (loss) for the best event of each act times the
probability of each events occurring.
C. Difference between the expected profit under certainty and the expected
opportunity loss.
D. Difference between the expected profit under certainty and the expected
monetary value of the best act under uncertainty.

32. D
? The expected value of perfect information.
 Choice-letter “d” is correct, the value of perfect information is the difference between
the expected value under certainty and the expected value of the best choice under
uncertainty. Choice-letter “a” is incorrect because it refers to the expected value
under certainty and not the expected value under perfect information. Choice-letter
“b” is incorrect because it refers to the expected value under uncertainty. Choice-
letter “c” is incorrect because the opportunity loss being mentioned may not refer to
the best choice under the environment of uncertainty.

33. A firm will produce product A or B. The total costs (TC) for both products can be
estimated by the equations
Product A: TC = P300,000 + (P23 x Sales volume)
Product B: TC = P100,000 + (P29 x Sales volume)
The firm believes there is a 20% chance for the sales volume of each product to
equal 10,000 units and an 80% chance they will both equal 20,000 units. The selling
price of product A is P42, and the selling price of product B is P40. The expected
profit from producing product B equals
A. P98,000 C. P390,000
B. P120,000 D. P680,000

33. D
? The expected profit in producing product B.
Chapter 22  Quantitative techniques in business 165

 Profit is the difference between sales and costs. The estimated sales is 18,000 units:
10,000 units x 20% = 2,000 units
20,000 units x 80% = 16,000 “
Estimated sales 18,000 units
Product B has a unit sales price of P40 and a unit variable cost of P29, therefore,
has a unit contribution margin of P11. The profit from producing product B shall be:
Contribution margin (18,000 units x P11) P198,000
- Fixed costs 100,000
Profit P 98,000

Questions 34 through 36 are based on the following information. A beverage stand


can sell either soft drinks or coffee on any given day. If the stand sells soft drinks and
the weather is hot, it will make P2,500; if the weather is cold, the profit will be P1,000.
If the stand sells coffee and the weather is hot, it will make P1,900; if the weather is
cold, the profit will be P2,000. The probability of cold weather on a given day at this
time is 60%.

34. The expected payoff for selling coffee is


A. P1,360 C. P3,900
B. P2,200 D. P1,960

34. D
? The expected payoff for selling coffee.
 The expected payoff is the same as the expected value or the expected profit.
Arranging the data given on the problem, we will have the following:
Weather
Product Hot Cold
Soft drinks P2,500 P1,000
Coffee 1,900 2,000
Probability 40% 60 %
The probability that the weather is cold is 60%, therefore the conditional
probability that the weather is hot is 40%. The expected payoff of selling coffee is
equal to the sum of its payoffs multiplied by their respective probabilities, such as:
Hot = P1,900 x 40% = P 760
Cold = P2,000 x 60% = 1,200
Expected value of selling coffee P1,960

35. The expected payoff if the vendor has perfect information is


A. P3,900
C. P1,960
B. P2,200
D. P1,950

35. B
? The expected value if the vendor has perfect information.
Chapter 22  Quantitative techniques in business 166

 Since there are only two probabilities of future event, either hot or cold, and there are
only two products soft drinks [SD] and coffee [C], then, there would only be two
feasible choices arising from two possible combinations, such as:
Future Event Expected Profits Better Choice Probability Expected Value
Hot SD, P2,500; C, P1,900 P2,500 40% P1,000
Cold SD, P1,000; C, P2,000 2,000 60% 1,200
Expected value with perfect information P2,200

36. Disregarding questions 34 and 35, if the probability of hot weather given a hot
weather forecast is 50%, how much would the vendor be willing to pay for the
forecast?
A. P 600 C. P1,000
B. P 300 D. P 800

36. B
? The value of perfect information if the hot weather forecast is 50%.
 The value of perfect information is the value that a reasonable person would be
willing to pay for the information. It is the difference between the expected value of
information under uncertainty (i.e, imperfect) and certainty.(i.e., perfect).
Expected value with uncertainty
Soft drinks [(P2,500 x 50%) + (P1,000 x 50%)] P1,750
Coffee [(P1,900 x 50%) + (P2,000 x 50%)] 1,950
Better choice under uncertainty (higher value) P1,950
Expected value with certainty:
Hot : Soft drinks = P2,500 x 50% P1,250
Cold : Coffee = P2,000 x 50% 1,000 2,250
Value of perfect information P 300

37. A firm must decide whether to introduce a new product A or B. There is no time to
obtain experimental information; a decision has to be made now. Expected sales can
be classified as weak, moderate, or strong. How many different payoffs are possible
in a decision tree under these circumstances?
A. 2 C. 5
B. 3 D. 6

37. D
? The number of possible payoffs in a decision tree.
 There are two products and three possible future events for each product. This
would mean 6 payoff tables (i.e., 2 x 3), or 6 combinations.

38. A company is evaluating the following information in an effort to determine which of


two products, A or B, it should manufacture during the coming year. Disregard
income tax effects:
Product A Product B
Expected Sales Probability (Units) Expected Sales Probability (Units)
7,000 .60 9,000 .75
8,000 .40 10,000 .25
1.00 1.00
Chapter 22  Quantitative techniques in business 167

Product A Product B
Selling price P20 P15
Variable cost per unit P10 P 8
Annual fixed manufacturing costs (all cash) P50,000 P 40,000
Annual company manufacturing expenses
(all cash) P20,000 P 20,000

The company is considering engaging a market research firm to better estimate its
sales. Assuming that the research firm can estimate sales with 100% accuracy, what
is the value of this perfect information to the company?
A. P750 C. P4,750
B. P2,100 D. P6,850

38. B
? The value of perfect information.
 The value of perfect information is the difference between the expected value under
imperfect information and expected value under perfect information, such as:
a. Expected value without perfect information:
Profit Probability Expected Value
Product A [(7,000 units x P10) – P70,000] P 0 60% P 0
[(8,000 units x P10) – P70,000] 10,000 40% 4,000
Total expected value of product A P4,000
Product B [( 9,000 units x P7) – P60,000] P 3,000 75% 2,250
[(10,000 units x P7) – P60,000] 10,000 25% 2,500
Total expected value of product B 4,750
Expected value without perfect information – higher value P4,750

b. Expected value without perfect information:


Payoff mix Profit Expected
A B A B Better Choice Joint Probability Value
7,000 9,000 P 0 P 3,000 B, P 3,000 (60% x 75%) = 45% P1,350
7,000 10,000 0 10,000 B, 10,000 (60% x 25%) = 15 1,500
8,000 9,000 10,000 3,000 B, 10,000 (40% x 75%) = 30 3,000
8,000 10,000 4,000 10,000 A or B, 10,000 (40% x 25%) = 10 1,000
Expected value with perfect information P6,850

c. Value of perfect information


Expected value with perfect information P6,850
- Expected value without perfect information 4,750
Value of perfect information P2,100

39. Philip Enterprises, distributor of compact disks (CDs), is developing its budgeted cost
of goods sold for 2006. Philip has developed the following range of sales estimates
and associated probabilities for the year:
Sales Estimate Probability
P60,000 25%
85,000 40
100,000 35
Chapter 22  Quantitative techniques in business 168

Philip’s cost of goods sold averages 80% of sales. What is the expected value of
Philip’s 2006 budgeted cost of goods sold?
A. P85,000 C. P68,000
B. P84,000 D. P67,200

39. B
? The expected value of the budgeted cost of goods sold.
 Cost of goods sold is the difference between sales and cost of goods sold. Sales
shall be equal to the sum of the sales estimates times their respective probabilities,
such as:
P60,000 x 25% = P15,000
85,000 x 40 = 34,000
100,000 x 35 = 35,000
Sales P84,000
Cost of goods sold shall be P67,200 (i.e., P84,000 x 80%).

Questions 40 through 43 are based on the following information. The College Honor
Society sells hot pretzels at the home football games. The pretzels are sold for P1.00
each, and the cost per pretzel is P.30. Any unsold pretzels are discarded because
they will be stale before the next home game. The frequency distribution of the
demand for pretzels per game is presented below.
Unit Sales Volume Probability
2,000 pretzels .10
3,000 pretzels .15
4,000 pretzels .20
5,000 pretzels .35
6,000 pretzels .20

40. The estimated demand for pretzels at the next home football game using an
expected value approach is
A. 4,000 pretzels. C. 5,000 pretzels.
B. 4,400 pretzels. D. Some amount other than those given.

40. B
? The expected demand using the expected value approach.
 The expected demand is computed as follows:
Unit sales volume Probability Expected Value
2,000 0.10 200
3,000 0.15 450
4,000 0.20 800
5,000 0.35 1,750
6,000 0.20 1,200
Expected demand 4,400

41. The estimated demand for pretzels at the next home football game using a
deterministic approach based on the most likely outcome is
A. 4,000 pretzels. C. 5,000 pretzels.
Chapter 22  Quantitative techniques in business 169

B. 4,400 pretzels. D. 6,000 pretzels.

41. C
? The estimated demand using the deterministic approach.
 The deterministic approach in estimating future event adopts the highest probable
probability as the most likely event to happen. On the data given, 5,000-unit level
has a 35% probability; hence, this is the estimated demand using the deterministic
demand.

42. The conditional profit per game of having 4,000 pretzels available but only selling
3,000 pretzels is
A. P1,800 C. P3,800
B. P2,100 D. Some amount other than those given.

42. A
? The conditional profit per game of having 4,000 units available for sale but only 3,000
units is sold.
 The conditional profit given a sales of 3,000 units and 4,000 units available for sale
shall be:
Sales (3,000 units x P1.00) P3,000
- Costs (4,000 units x P0.30) 1,200
Profit P1,800

43. The conditional profit per game of having 4,000 pretzels available and selling all
4,000 pretzels is
A. P1,200 C. P2,800
B. P2,100 D. P 800

43. C
? The conditional profit per game of having 4,000 units available for sale and all units
sold.
 The conditional profit given that all the available 4,000 units for sale are sold shall be
P2,800, as follows:
Sales (4,000 units x P1) P4,000
- Costs (4,000 units x P0.30) 1,200
Profit P2,800

44. The legal department of a firm prepared the decision tree below for a possible patent
infringement suit.
2,000,000
(.80) Win
(Trial)
(.20) Lose

(100,000)
No
(Settle)
(.50)Yes Yes 800,000
Chapter 22  Quantitative techniques in business 170

(Injunction) Yes 500,000


(.50)No (Settle) (.70)Win 2,000,000
No (Trial)
(.30)Lose (100,000)
FILE SUIT

(No gain/loss)

Based on the decision tree, the firm should


A. Not file the suit.
B. File suit; settle if injunction granted.
C. File suit; settle if injunction not granted.
D. Carry suit to trial.

44. D
? The action to be made based on the decision tree analysis.
 If the company files the suit, there are two possibilities, either an injunction is granted
or not. Either way, the firm may continue the suit to trial or enter into an amicable
settlement with the defendant. The expected value analysis, using the decision tree
data would be as follows:
Expected values
With injunction
Carry to trial {[(P2,000,000 x 80%) + (-P100,000 x 20%)] x 50%} P790,000
Settlement (P800,000 x 50%) 400,000
Without injunction
Carry to trial ({(P2.000,000 x 70%) + (-P100,000 x 30%)] x 50%} 685,000
Settlement (P500,000 x 50%) 250,000
The expected value of carrying the trial to suit is still multiplied by 50% because
there is a 50-50 chance of getting the injunction or not. The best alternative having
the highest expected value is when an injunction is received and the suit is carried to
trial.
Among the choice given, the best answer is choice-letter “d” because the
company would be better off carrying the suit to trial regardless of whether an
injunction is granted or not. These alternatives have higher expected values than
entering into an amicable settlement with the defendant. Choice-letter “a” is incorrect
because the company would benefit regardless of the outcome of the suit. Choice-
letters “b” and “c” are incorrect because entering into a settlement would mean a
lesser benefit to the company either an injunction is ordered by the court or not.

45. The modeling technique to be used for situations involving a sequence of events with
several possible outcomes associated with each event is
A. Queuing theory. C. The critical path method.
B. Dynamic programming. D. Decision tree analysis.

45. D
? The modeling technique to be used for situations involving a sequence of events with
several possible outcomes associated with each event.
Chapter 22  Quantitative techniques in business 171

 Choice-letter “d”, decision tree analysis is the best technique to be used for situations
involving a sequence of events with several possible outcomes associated with each
event. A decision tree analysis identifies the possible outcomes of each event
together with their respective probabilities and permits the determination of expected
value for each event.
Choice-letter “a” is incorrect because queuing theory is used to balance the cost
of providing a service facility with the savings derived from the reduced cost of
waiting. Choice-letter “b” is incorrect because dynamic programming is an approach
to solving problems, not a particular algorithm. It analyses a part of a large
mathematical model in such a way that once the problem in the part is solved, the
optimal solution to the overall model is done. Choice-letter “c” is incorrect because
the critical path method deals with the optimization of project cost through the
scientific way of managing project activities by reducing project time and cost.

46. Which one of the following statements does not apply to decision tree analysis?

A. The sum of the probabilities of the events is less than one.


B. All of the events are mutually exclusive.
C. All of the events are included in the decision.
D. The branches emanate from a mode from left to right.

46. A
? A statement that does not apply to decision tree analysis.
 Choice-letter “a” is a false statement with regard to decision tree analysis and is the
correct answer. The sum of all conditional probabilities of the events is always equal
to 1.0. Choice-letters “b”, “c”. and “d” are correct statements with regard to decision
tree analysis. It assumes that ell events are mutually exclusive, all of the events are
included in the decisions, and branches in the analysis emanate from the left to the
right.

47. The modeling technique to be employed in a situation involving a sequence of events


with several possible outcomes associated with each event is
A. Network analysis. C. Monte Carlo simulation.
B. Decision tree analysis. D. Linear programming.

47. B
? The modeling technique to be used for situations involving a sequence of events with
several possible outcomes associated with each event.
 Choice-letter “b” is correct because decision tree analysis identifies the possible
outcomes of each event together with their respective probabilities thereby permitting
the determination of expected value for each event. Choice-letter “a” is incorrect
because network analysis is used in project management with the end-in-view of
minimizing project time and cost. Choice-letter “c” is incorrect because Monte Carlo
simulation is a technique for experimenting with mathematical models using a
computer, which generates random values for each variable. Choice-letter “d” is
incorrect because linear programming is a technique used in optimizing the benefit
given by scarce resources by setting the objective function within the constraints of
limited resources.
Chapter 22  Quantitative techniques in business 172

48. The use of a decision tree is appropriate for decision making under conditions of
A. Uncertainty and risk.
B. Uncertainty and subjective likelihood.
C. Certainty.
D. Risk.

48. D
? The condition under which the use of decision tree is appropriate.
 Decision tree analysis considers the possible outcomes of various events with a
certain degree of certainty. It does not operate on a condition of uncertainty where
the outcome of a future event is not reasonably determinable. When the outcome of
a future is reasonably determinable, then risk analysis and expected value analysis
may come into use. Choice-letter “d”, under the condition of risk, is correct.
Choice-letters “a” and “b” are incorrect because both considers uncertainty as an
answer. Choice-letter “c” is incorrect because when there is condition of certainty,
then there is no need for a decision tree where the possible outcomes are identified
with their respective probabilities and payoffs. Again, decision tree operates under
some degree of uncertainty.

Forecasting methods
49. To facilitate planning and budgeting, management of a travel service company wants
to develop forecasts of monthly sales for the next 24 months. Based on past data,
management has observed an upward trend in the level of sales. There are also
seasonal variations with high sales in June, July and August, and low sales in
January, February and March. An appropriate technique for forecasting the
company’s sales is
A. Time series analysis. C. Linear programming.
B. Queuing theory. D. Sensitivity analysis.

49. A
? The appropriate technique for forecasting the company’s sales for the next 24
months based on past data with seasonal variations in some months and a recent
upward trend in the level of sales.
 Choice-letter “a” is correct because times series analysis is also used in forecasting
based on the regression equation Y = a + bx. This equation becomes a time series
equation when the value “x” is expressed in terms of time such as months, weeks,
years, quarters, and the like. Choice-letter “b” is incorrect because queuing theory is
not used in forecasting but on maximizing the benefit of providing the service with the
cost of waiting, or vice-versa. Choice-letter “c” is incorrect because linear
programming is used in determining the optimal mix to maximize profit given the
constraints in the environment. Choice-letter “d” is incorrect because sensitivity
analysis is geared more in determining the effects of the changes in variables in a
given mathematical model.

50. What are the four components of a time series?


A. Trend, cyclical, seasonal and irregular.
B. Alpha, cyclical, seasonal and irregular.
Chapter 22  Quantitative techniques in business 173

C. Alpha, cyclical, seasonal and repetitive.


D. Trend, cyclical, seasonal and repetitive.

50. A
? The four components of the time series.
 Choice-letter “a” is correct, the four components of the time series are the secular
trend, cyclical variation, seasonality, and irregularity (or random variation). Choice-
letter “b” is incorrect because alpha factor is not a component of time series. Choice-
letters “c” and “d” are not incorrect because repetitive factor is a subordinate variable
within the irregularity of observation.

51. The four components of time series data are secular trend, cyclical variation,
seasonality and random variation. The seasonality in the data can be removed by
A. Multiplying the data by a seasonality factor.
B. Ignoring it.
C. Taking the weighted average over four time periods.
D. Subtracting a seasonality factor form the data.

51. C
? The technique that may be used in removing the effects of seasonality variation in the
data.
 Seasonal variations are common in most businesses. A seasonal variation may be
included in the forecast by using a seasonal index. Alternatively, a seasonal variation
can be removed from data by using a weighted average of several time periods
instead of data from individual period, hence, choice-letter “c” is correct.
Choice-letters “a” and “d” are incorrect because they simply make adjustments
for seasonality factor and not removing it. Choice-letter “b” is incorrect because you
cannot ignore the seasonality variation in making a forecast.

52. An internal auditor for a large automotive parts retailer wishes to perform a risk
analysis and wants to use an appropriate statistical tool to help identify stores that
are at variance with the majority of stores. The most appropriate statistical tool to use
would be
A. Linear time series analysis.
B. Cross-sectional regression analysis.
C. Cross tabulations with chi square analysis of significance.
D. Time series multiple regression analysis to identify changes in individual stores
over time.

52. B
? The most appropriate statistical tool of risk analysis to identify stores that are at
variance with the majority of stores.
 Choice-letter “b” is correct because cross-sectional regression analysis compares the
regression lines of different stores in a single graph and would determine the stores
that are at variance with the majority of the stores. Choice-letter “a” is incorrect
because it strictly refers to linear time series analysis where time is the only
independent variable considered and that such would give only an interpretation for a
single store without impressions about its variance with the other stores. Choice-
Chapter 22  Quantitative techniques in business 174

letter “c” is incorrect because making a chi-square analysis would not give an
analysis of variance of a store with another but rather would help in interpreting
whether a null hypothesis is acceptable or not. Choice-letter “d” is incorrect because
that changes to be identified is not within a store over time but that of various stores
at a time.

53. The moving-average method of forecasting


A. Is a cross-sectional forecasting method.
B. Regresses the variable of interest on a related variable to develop a forecast.
C. Derives final forecast by adjusting the initial forecast based on the smoothing
constant.
D. Includes each new observation in the average as it becomes available and
discards the oldest observation.

53. D
? A statement regarding moving-average method of forecasting.
 Choice-letter “d” is correct because a moving average includes the recent data in the
computation of average observation and discards the old observation. The new
average observation equals the value of the immediate past observations + 1 divided
by the number of defined observation. Choice-letter “a” is incorrect because a cross-
sectional analysis measures a variance of one event with another events. Choice-
letter “b” is incorrect because a moving average technique does not regress a
variable of interest on a related variable to develop a forecast but instead regresses
using the new average observation determined including the recent data. Choice-
letter “c” is incorrect because using a smoothing constant to arrive at a final forecast
is a weighted average technique.

54. As part of a risk analysis, an auditor wishes to forecast the percentage growth in next
month’s sales for a particular plant using the past 30 month’s sales results.
Significant changes in the organization affecting sales volumes were made within the
last 9 months. The most effective analysis technique to use would be
A. Unweighted moving average. C. Queuing theory.
B. Exponential smoothing. D. Linear regression analysis.

54. B
? The most effective analysis technique in forecasting the percentage growth in the
next month’s sales over a data of 30 months given significant changes in the last 9
months.
 Choice-letter “b” is correct because exponential smoothing a version of weighted
average method where the latest data is given an assigned percentage and the last
average observation is also given an assigned percentage equal to 1 minus the
assigned percentage to the latest data. Choice-letter “a’ is incorrect because
unweighted moving average simply considers the latest data and therefore would
make the past data irrelevant. Choice-letter “c” is incorrect because queuing theory
is a technique applied not on the forecasting area but on balancing the cost and
benefits of providing a service facility and waiting. Choice-letter “d” is incorrect
because linear regression analysis places equal emphasis on all the data presented
without giving exceptions on significant variations.
Chapter 22  Quantitative techniques in business 175

Simulation / Sensitivity analysis / Scenario analysis / Predictive value analysis


55. In evaluating projects, a popular approach to recognize uncertainty about individual
items and to obtain an immediate financial estimate of the consequences of possible
predicting errors is?
A. Exponential analysis. C. Learning curve analysis.
B. Sensitivity analysis. D. Expected value analysis. (rpcpa)

55. B
? A popular approach to recognize uncertainty and obtain financial estimate of the
consequences of possible predicting errors.
 Choice-letter “b”, sensitivity analysis, is the correct answer. Sensitivity analysis
measures the possible outcome in a given environment if a particular event
happened or not happened. This model also uses probability principles and is a
popular model to recognize uncertainty and estimate the consequences of possible
predicting errors.
Choice-letter “a” is incorrect because exponential analysis estimates the outcome
of an event given a particular strict condition that is to be followed. Choice-letter “c”
is also incorrect because learning curve analysis relates to changes in the
productivity of manpower as production doubles and is insignificantly related to
uncertainty. Choice-letter “d” is also incorrect because expected value analysis
estimates the best value of a future event given the surrounding probability of
occurrences of each future event.

56. A company is simulating the actions of a government agency in which 50% of the
time a recall of a product is required, 40% of the time only notification of the buyer
about a potential defect is required, and 10% of the time no action on its part is
required. Random numbers of 1 to 100 are being used. An appropriate assignment
of random numbers for the recall category would be
C. 1-40
C. 61-100
D. 40-90
D. 11-60

56. D
? An appropriate assignment of random numbers for the recall category
 The recall category is 50% of the total probability. Given a total of 100 random
numbers, the recall category should be assigned a total of 50 random numbers (i.e.,
100 x 50%). Among the choices, only choice-letter “d” allocates a total of 50
numbers, from 11-60. Choice-letter “a” has only 10 random numbers, choice-letter
“b” has 51 random numbers, and choice-letter “c” has 40 random numbers.

57. Quick Response Plumbing (QRP), a wholesale distributor, supplies plumbing


contractors and retailers throughout the Northeast on a next-day delivery basis. QRP
has a centrally located warehouse to accept receipts of plumbing supplies. The
warehouse has a single dock to accept and unload railroad freight cars during the
night. It takes five hours to unload each freight car. QRP’s prior records indicate that
the number of freight cars that arrive in the course of a night range from zero to five
Chapter 22  Quantitative techniques in business 176

or more, with no indicated pattern of arrivals. If more than two freight cars arrive on
the same night, some freight must be held until the next day for unloading. QRP
wants to estimate the wait time when more than two freight cars arrive in the same
night. The appropriate technique to analyze the arrival of freight cars is
A. Integer programming. C. Monte Carlo simulation.
B. Linear programming. D. Regression analysis.

57. C
? The appropriate technique to analyze the arrival of freight cars given no indicated
pattern of arrivals.
 Since there is no given pattern as to the arrival of the freight cars, the use of random
variables would be of great help. The Monte Carlo simulation method is often used
to generate the individual values for a random variable. Random values are selected
for each variable (based on the probability distribution of each variable) and then the
value of the solution is calculated in a given quantitative model under uncertainty. If
this process is performed many times, the distribution of results from the model will
be obtained. Choice-letter “c” is correct.
Choice-letter “a” is incorrect because integer programming is a variation of linear
programming that concerns problems in which some variables (e.g., problems) are
not continuous. Integer problems are also known as discrete models because the
variables take on discrete, noncontinuous values. Choice-letter “b” is incorrect
because linear programming concerns with optimizing the return of limited resources
and not on the probability that an event may occur or not. Choice-letter “d” is
incorrect because regression analysis is used to predict the future based on the trend
provided by the past data.

58. When simulating with the Monte Carlo technique, the average simulated demand
over the long run should approximate the
A. Actual demand. C. Sampled demand.
B. Real demand. D. Expected demand.

58. D
? The one that approximates the average simulated demand over the long-run using
the Monte Carlo technique.
 Choice-letter “d” is correct because expected demand or expected value of demand
represents the sum of the future values multiplied by their respective probability. This
approximates the average simulated demand over the long-run. Choice-letter “a” is
incorrect because actual demand is not an approximation. Choice-letter “b” is
incorrect because real demand may refer to the existence of demand. Choice-letter
“c” is incorrect because sampled demand may refer to the presence of a demand
based on selected samples which may not represent the simulated demand over the
long run.

59. A widely used approach that mangers use to recognize uncertainty about individual
items and to obtain an immediate financial estimate of the consequences of possible
prediction errors is
A. Expected value analysis. C. Sensitivity analysis.
B. Learning curve analysis. D. Regression analysis.
Chapter 22  Quantitative techniques in business 177

59. C
? An approach that provides an immediate financial estimate of the consequences of
possible prediction error.
 Choice-letter “c” is correct because sensitivity analysis (or simulation or scenario
analysis) predicts the possible outcome of an event after considering the changes in
the variables affecting the event. This approach gives an analyst the financial
estimates that may be of consequence, either profit or loss, to variable changes.
Choice-letter “a” is incorrect because expected value analysis measures the
estimated value of an event given its assigned probability rates but does not measure
the effects of the changes in the consequence of an event. Choice-letter “b” is
incorrect because learning curve analysis relates to reduction in the labor to be
incurred for every doubling of output. Choice-letter “d” is incorrect because
regressions analysis estimates financial estimate based on the trend of past
observations.

60. Through the use of decision models, managers thoroughly analyze many alternatives
and decide on the best alternative for the company. Often the actual results
achieved from a particular decision are not what was expected when the decision
was made. In addition, an alternative that was not selected would have actually been
the best decision for the company. The appropriate technique to analyze the
alternatives by using expected inputs and altering them before a decision is made is
A. Expected value analysis.
B. Linear programming.
C. Program Evaluation Review Technique (PERT).
D. Sensitivity analysis.

60. D
? The appropriate technique to analyze alternatives by using expected inputs and
altering them before a decision is made.
 Sensitivity analysis measures the outcome in a mathematical model by changing a
variable(s) in the model. This provides a decision maker to predict the possible
outcome of various decisions to be made, hence, giving him the chance to determine
the best payoff among the alternatives. Choice-letter “d”, sensitivity analysis is
correct.
Choice-letter “a” is incorrect because expected value analysis relies more on
conditional probability distribution to measure the result of future event and not much
on the effects of the changes in the variables affecting the future event. Expected
value analysis complements sensitivity analysis. Choice-letter “b” is incorrect
because linear programming identifies the best use of scarce resources. Choice-
letter “c” is incorrect because program evaluation review technique (PERT) organizes
the best scheduling of project activities to reduce time and costs.

Linear programming
61. The El Kulagtit Company has two products, Product X and Product Y, that it
manufactures though its production facilities. The contribution margin for Product X is
P15.00 per unit, whereas Production Y’s contribution is P25.00. Each product uses
Materials A and B. Product X uses 3 pounds of Material A and Product Y used 6
pounds. Product X requires 6 feet of Material B and Product Y uses 4 feet. The
Chapter 22  Quantitative techniques in business 178

company can only purchase 600 pounds of Material A and 880 feet of Material B.
The optimal mix of products to manufacture is
Product X Product Y
A. 146 units 0 units
B. 0 units 100 units
C. 120 units 40 units
D. 40 units 120 units

61. C
? The optimal mix of products to manufacture.
 The company produces two products, X and Y, with the following related production
data and constraints:
Product Material A Material B Profit per unit
X 3 lbs. 6 ft. P15.00
Y 6 lbs. 4 ft. 25.00
Available resource 600 lbs. 880 ft.

The following equations may be developed from the data presented:

Assume : X = Product X, and Y = Product Y


Objective function : Max profit = 15X + 25Y
If (subject to) : 3X + 6Y ≤ 600
6X + 4Y ≤ 880
X, Y ≥ 0
a.) Using the equation method in solving for the unknown values of X and Y, the
inequality equations are first converted to equalities and by elimination
method we will have:
Equation 1 : [ 3X + 6Y = 600 ] -2
Equation 2 : 6X + 4Y = 880
-6X - 12Y = -1,200
- 8Y = - 320
Y = -320 / -8 = 40
The negative 2 (e.g., -2) is the eliminating factor and is determined by getting
the quotient of the “X” variable in the two equations (i.e., 6/3). This is done to
eliminate “X” and easily solve for the value of “Y”. Since the value “Y” is
already made known to be 40, then, by substitution, the value of “X” shall be:
3X + 6Y = 600
3X + 6(40) = 600
3X = 600 – 120 = 120

b.) Using the graphical method, the initial values of variables “X” and “Y” in the
two equations may be determined as follows:
Equation 1 : 3X + 6Y = 600 if X = 0; then, Y = 100
Y = 0; then, X = 200
Equation 2 : 6X + 4Y = 880 if X = 0; then, Y = 220
Y = 0; then, X = 147
By plotting the values of the variables in the graph, we will have:
Chapter 22  Quantitative techniques in business 179

Y
Eq 2

250

200

Eq 1
150

100

50

0 X
50 100 150 200 250 300

The covered area (or shaded portion) of the equations are made downwards
because of the original state of the equations in their inequality versions less
than or equal to (i.e., ≤). All the production mix within the shared covered
area on the graph would result to profit, therefore, all those covered on the
shared shaded area would be a feasible point. However, the production mix
of products X and Y would result to highest profit on the corner areas (or
corner points or vertices). The corner points and their estimated profits are:
Corner points
X Y Profit = 15X + 25Y
147 0 15 (147) + 25 (0) = P2,205
0 100 15 (0) + 25 (100) = P2,500
120 40 15 (120) + 25(40) = P2,800
The corner point, X = 120 and Y = 40, produces the highest profit at P2,800,
then that will be best production mix to maximize the objective of the firm.

Questions 62 to 64 are based on the following information. Phy Manufacturing


Corporation uses the following model to determine its product mix for metal (M) and
scrap metal (S).
Max Z = P30M + P70S
If: 3M + 2S < 15
2M + 4S < 18

62. These mathematical functions are an example of a(n)


A. Simulation model. C. Economic order quantity model.
B. Linear programming model. D. Present value model.

62. B
? The mathematical model that uses the given mathematical functions.
Chapter 22  Quantitative techniques in business 180

 The mathematical functions (e.g., or equations) expressed in the problem predicate


that of the linear programming model. In this model, an objective function is
established subject to several constraints. Hence, choice-letter b” is correct.
Choice-letter “a” is incorrect because simulation model measures the effects of
changes in variables to the results of the mathematical model considered. Choice-
letter “c” is incorrect because economic order quantity model determines the optimal
order size of inventory to minimize the total relevant costs of ordering and carrying.
Choice-letter “d” is incorrect because the present value method determines the
present value of cash inflows using the present value factors of a chosen discount
rate less the cost of investment to give way to the net present value.

63. A quantitative technique used for selecting the combination of resources that
maximized profits or minimized costs is
A. Curvilinear analysis. C. Linear programming.
B. Queuing theory. D. Dynamic programming. (rpcpa)

63. C
? A quantitative technique used for selecting the combination of resources that
maximizes profit or minimizes costs.
 Linear programming is a technique used to maximize revenue or profit function, or
minimize a cost function, subject to constraints (e.g., limited/scarce resources,
maximum/minimum levels of production, performance, etc.). It is used in planning
resource allocations such as selecting a product mix, blending a chemical product,
scheduling flight crews, assigning jobs to machines, determining transportation
routes, and the like. Linear determining is applicable if there are at most two (2)
constraining variables (choice-letter “c” is correct).
Choice-letter “a”, curvilinear analysis has not found regular use in the field of
advisory services due to its advanced complexity and applications which run counter
to the simplistic assumptions used in the cost-volume-profit analysis and other short-
term analytical techniques. Curvilinear analysis is applied on studies outside the
relevant range where the linearity assumption does not accurately reflect the
observation over the long-run. This technique assumes waves of possibilities and
not straight line of possibilities.
Choice-letter “b”, queuing theory (waiting-line) is a group of mathematical models
for systems involving waiting line consists of a queue (or waiting line) and a service
facility. Its main concern is to reduce the waiting time and maximize the use of
service facility. It is applied on bank teller windows, grocery checkout counters,
highway tool booths, decks for ships, airport holding patterns, and the like.
Choice-letter “d”, dynamic programming is an approach for solving problems, not
a particular algorithm. It divides a large mathematical, model, into smaller more
manageable pieces that once the smaller problems have been solved, the result is
the optimal solution to the overall model or problem. Its uses include inventory rules,
capital budgeting for new ventures, systematic search for scarce resources, finding
the shortest route through a network, production scheduling in the face of a
fluctuating demand, spare parts level determinations, and the like.

64. A transportation Model is a special case of


A. Dynamic programming model. C. Linear programming model.
Chapter 22  Quantitative techniques in business 181

B. PERT/CPM. D. Economic order quantity.


(rpcpa)

64. C
? The correct classification of a transportation model.
 Transportation model is a special case of a linear programming model where
materials or objects are moved from one warehouse or location to another and
determining the optimum route and use of the resources available. Choice-letter “c”
is correct.
Choice-letter “a” is incorrect because dynamic programming relates to the use of
many variables that may or may not subject to constraints with the end-in-view of
maximizing results. Choice-letter “b” relates to project scheduling to effectively
complete projects on time and avoid excessive costs of completing a project.
Choice-letter “d” is also incorrect because it relates to the optimum level of order size
to be made in relation to purchasing functions of materials and inventories.

65. Given the basic equations for maximization of profits in linear programming model,
what quantitative techniques would generally be employed to arrive at an optimal
solution?
A. Markov analysis. C. Monte-Carlo analysis.
B. Regression analysis. D. Simplex-method analysis.
(rpcpa)

65. D
? A model in linear programming.
 Linear programming is used in determining the best (optimum) use of scarce
resources to maximize profit either through maximization of revenue or minimization
of costs. Linear programming also uses the simplex method and transportation table
in its analysis.
Choice-letter “a”, Markov analysis, is used in cases where the probability of the
occurrence of a future state depends only on the current state. A characteristics of
the Markov process is that the initial state matters less and less as time goes on
because the process will eventually reach its steady state. Choice-letter “b”,
regression analysis, is used to find trend lines and develop models based on the
association of variables. Choice-letter “c”, Monte Carlo analysis, is used in simulation
analysis where individual values are determined for a random variable. A random
number generator is used to produce numbers with a uniform probability distribution
(equal likelihood of occurrence). Then, it transforms the random numbers into values
consistent with desired distribution.

66. The two inequality functions are


A. Contributions. C. Objectives.
B. Shadow points. D. Constraints.

66. D
? The description to the two inequality functions.
 The two inequality functions represent the limitation on resources or constraints in the
operations. Choice-letter “a” is incorrect because contribution refers to the difference
Chapter 22  Quantitative techniques in business 182

between sales and variable costs and expenses. Choice-letter “b is incorrect


because shadow points refer to the area on the graph where the feasible option
locates. Choice-letter “c” is incorrect because the objective function refers to the
profit maximization.

67. The point at which M=3 and S=3 would


A. Minimize cost. C. Be a feasible point.
B. Lie in a corner. D. Be the optimal solution point.

67. C
? The point at which M=3 and S=3.
 The point at which M=3 and S=3 is one of the feasible corners, computed as follows:
Equation 1 : 3M + 2S = 15
M = (15-2S) / 3
M = 5 – 2/3S
Equation 2 : 2M + 4S = 18
2(5 – 2/3S) + 4S = 18
10 – 1.33S + 4S = 18
S = 5/2.67
S = 3
To compute for the value of M, by substitution, we have:
3M + 2S = 15
3M + 2(3) = 15
3M + 6 = 15
3M = 15 - 6
M = 9/3
M= 3

68. Phil-Fuji Company manufactures two types of electronic components, both of which
must pass through the Assembly and Finishing Departments. The following
constraints apply:
Contribution
Unit selling per Hours required per unit
Product price component Assembly Finishing
Component 818 P120 P30 3 4
Component 810 180 45 4 6

Demand for Component 818 far exceeds the company’s capacity, but the company
can only sell 60 units of components 810 each week. Workers in the Assembly
department work a total of 200 hours per week, and workers in the Finishing
department work a total of 250 hours per week. The company wants to know how
many units of each components to produce to maximize profit. If X represents the
number of units of Component 818 and Y represents the number of units of
Component 810, the objective function would be
A. Maximize 120X + 180Y. C. Minimize 90X + 135Y.
B. Maximize 30X + 45Y. D. Minimize 30X + 45Y.
(rpcpa)
Chapter 22  Quantitative techniques in business 183

68. B
? The objective function in a linear programming model.
 Linear programming is used to determine the optimum use of limited/scarce
resources to maximize profit through maximization of revenue or minimization of
costs. Therefore, the objective function is maximization of revenue or minimization of
costs.
The profit is represented by the contribution margin rate. Components 818 and
810 have P30 and P45 unit contribution margin respectively. If X = component 818
and Y = component 810, then, the objective function is, Profit = 30X + 45Y.

69. Quantitative technique used for selecting the combination of resources that
maximizes profits or minimizes profits or minimizes cost is:
A. Linear programming. C. Economic order quantity.
B. PERT-CPM. D. Correlation analysis.
(rpcpa)

69. A
? A quantitative technique used for selecting the combination of resources that
maximizes profit or minimizes cost.
 `Choice-letter “a” is the correct answer. Linear programming is used in optimizing the
use of resources if there are at most two limitations with the end view of maximizing
revenue or minimizing cost.
Choice-letter “b” is incorrect because PERT-CPM are network models designed
to chart the fastest possible way to complete a project at the least or most
economical cost involved. Choice-letter “c” is incorrect because economic order
quantity (EOQ) is used to determine the most optimum order size where the total
inventory cost of ordering and carrying is at their minimum. Choice-letter “d” is
incorrect because correlation analysis pertains more in knowing the relationship of
two variables (say: X and Y), whether positive or negative, and at what degree of
relationship.

70. Linear programming is used most commonly to determine


A. The fastest timing.
B. The best use of scarce resources.
C. The most advantageous prices.
D. The mix of variable that will result in the largest quantity. (rpcpa)

70. B
? The most common use of linear programming.
 Linear programming, as a quantitative technique is most commonly used to
determine the optimum use of limited/scarce resources to increase profit either
through maximization of revenue or minimization of costs (choice-letter “b”).
Choice-letter “a”, refers to PERT/CPM; choice-letter “c” refers to pricing models;
choice-letter “d” is irrelevant.

71. A mechanized system of handling parts from one assembly line to another is being
contemplated by the Moonbeam Co. The technical evaluation indicated that the
Chapter 22  Quantitative techniques in business 184

system will reduce labor and waiting time costs substantially. An assessment has to
be made of cost/benefit relationships including the effects of interest. The most
relevant quantitative technique to evaluate the project is:
A. Regression analysis. C. Time adjusted rate of return analysis.
B. PERT-CPM. D. Payback period analysis. (rpcpa)

71. C
? Considering the effects of interest, the most relevant quantitative technique to
evaluate whether a production process shall be changed or not.
 The decision as to whether the production process shall be changed to a mechanized
system or not should pass the criteria of the cost –benefit analysis. In capital
budgeting, the project evaluation techniques (i. e., payback period, accounting rate or
return, discounted cash flows0 are used to determine the acceptability and priority of
a proposed project. However, in this problem, the effects of interest are to be
considered.
Choice-letter “c” is correct because the time-adjusted rate of return (or internal
rate of return) is a criterion that is used to measure whether the benefit that could be
derived from the decision (e.g., present value of cash inflows) is greater or smaller
than the cost of the decision (e.g., cost of investment) with consideration to the time
value of money or effects of interest. If the internal rate of return is higher than the
required rate or return (e.g., cost of capital, hurdle rate), the project is acceptable,
otherwise it is not.
Regression analysis (choice-letter “a”) is used to study the classification,
behavior, and trend of revenues and costs. PERT-CPM (choice-letter “b”) is a
technique used to determine the best scheduling and sequence of activities to
complete a project at the best possible time an the least possible cost. Choice-letter
“d” is also a cost-benefit criterion but it does not consider the time value of money or
the effects of interest rate.

72. Linear programming models are mathematical techniques in which an objective


function is maximized or minimized subject to constraints. These constraints must; be
fully specified before a linear programming problem can be solved, and generally
describe:
A. Costs. C. Inefficiencies.
B. Resources. D. Dependent variables. (rpcpa)

72. B
? The description of constraints in the linear programming model.
 Constraints in linear programming are also known as limited resources.
Linear programming is mathematical technique used to maximize revenue (profit
function) or minimize cost (cost function) subject to constraints. Examples of these
constraints are scarce (limited) resources, maximum or minimum level of production
or performance.

73. In a linear programming maximization problem for business problem solving, the
coefficient of the objective function usually are
A. Usage rates for scarce resources.
B. Profit based on allocations of overhead and all indirect costs.
Chapter 22  Quantitative techniques in business 185

C. Variable costs.
D. Marginal contributions per unit.
(rpcpa)

73. D
? The usual coefficient of the objective function in the linear programming maximization
problem.
 The objective function of a linear programming maximization problem is usually to
maximize profit. And profit is expressed by the contribution margin per unit in the
marginal costing analysis. Hence, choice-letter “d” is correct.
Choice-letters “a”, “b”, and “c” are all incorrect because they are unreasonable
basis of maximizing the profit of a firm.

74. An investment company is attempting to allocate its available funds between two
investment alternatives, stocks and bonds, which differ in terms of expected return
and risk. The company would like to minimize its risk while earning an expected
return of at least 10% and investing no more than 70% in either of the investment
alternatives. An appropriate technique for allocating its funds between stocks and
bonds is
A. Linear programming. C. Differential analysis.
B. Capital budgeting. D. Queuing theory.

74. A
? An appropriate technique for allocating limited funds between stocks and bonds.
 Allocation of limited funds comes within the purview of linear programming where a
limited resource is optimized in its use. Choice-letter “a” is correct because the
limited funds should be allocated between stocks and bonds where investment in a
security is no more than 70% of the total available funds.
Choice-letter “b” is incorrect because capital budgeting relates to the
determination of best possible investments given a particular mix of financing, and is
not a technique used in optimizing the use of limited resource (i.e., funds). Choice-
letter “c” is incorrect because differential analysis is applicable in getting the
difference in each alternative and in the process it also determines the alternative the
gives the best benefit. Choice-letter “d” is incorrect because queuing theory applies
to balancing the cost of providing a service with the cost of waiting if the service is not
provided.

75. Linear programming is an operations research technique that allocates resources.


Mathematical expressions are used to describe the problem. The measure of
effectiveness that is to be maximized or minimized is the
A. Constraints. C. Objective function.
B. Set of decision variables. D. Derivative of the function.

75. C
? The measure of effectiveness that is to be minimized or maximized in a linear
programming method.
 Choice-letter “c” is correct because the objective function measures the results of
using limited resources. Choice-letter “a” is incorrect because constraints refer to
Chapter 22  Quantitative techniques in business 186

limited resources. Choice-letter “b” is incorrect because a set of decision variables


refers to the expression of recognizing the objective functions and resource
limitations. Choice-letter “d” is incorrect because it does not have relevant use in
linear programming.

76. The constraints in a linear programming model are


A. Included in the objective function. C. Scarce resources.
B. Costs. D. Dependent variables.

76. C
? The constraints in a linear programming model.
 Choice-letter “c” is correct because the constraints in a linear programming model are
defined as scarce resources. It is the limitations where the business shall undertake
its activities in order to maximize the use of its resources.
Choice-letter “a’ is incorrect because constraints are not included in the objective
functions. Choice-letter “b” is incorrect because costs are not the only expressions of
constraints, and therefore, is an inferior choice. Choice-letter “d” is incorrect because
dependent variables are those where their values are determined by a change in an
independent variable.

77. United Industries manufactures three products at its highly automated factory. The
products are very popular, with demand far exceeding the company’s ability to supply
the marketplace. To maximize profit, management should focus on each product’s
A. Gross margin. C. Contribution margin.
B. Segment margin. D. Contribution margin per machine hour.

77. D
? The item to which management should focus to maximize profit on each product.
 The profit of each product is expressed by its contribution margin per limited
resource, which is indicated in the given choices as the machine hour. Since the
demand in the market place of each product far exceeds the supply, the product that
provides the highest contribution margin per hour should be prioritized as to
production and sales. Choice-letter “d” is correct.
Choice-letter “a” is incorrect because gross margin does not reflect the
profitability of a product per limited resource but rather the profitability of a product
based on its sales. Choice-letter “b” is incorrect because segment margin, just like
the gross margin, only determines the profitability of a product based on the number
of units sold. Choice-letter “c” is also incorrect because contribution margin per se
refers to profit contributed per unit of product sold and not profit based on the
maximization of scarce resource.

78. To solve a linear programming problem, slack, surplus and artificial variables must be
employed. A slack variable represents
A. Opportunity costs. C. Outside variables with high cost.
B. Unused capacity. D. The variable with the most negative value.

78. B
? The item represented by the slack variable.
Chapter 22  Quantitative techniques in business 187

 A slack variable (“S”) is used in an inequality function “less than or equal to”. To
equalize the equation characterize by the inequality function “less than or equal to”, a
slack variable is added on the left side of the equation to serve as a buffer for an
unknown value of a variable. This unknown value may refer to the unused capacity,
hence, choice-letter “b:” is correct.
Choice-letter “a” is incorrect because opportunity cost is not an unknown variable
that my be used as a buffer in equalizing an equation but rather a benefit foregone in
favor of choosing another alternative. Choice-letter “c” is incorrect because a slack
variable is an internal one and not coming from outside. Choice-letter “d” is incorrect
because a slack variable, which normally represents an unused capacity, is positive
in its expression.

79. Sensitivity analysis in linear programming is used to


a. Test the accuracy of the parameters.
b. Develop the technological matrix.
c. Determine how the optimal solution will react to changes in parameters.
d. Develop objective function coefficients.

79. C
? The use of sensitivity analysis in linear programming.
 Sensitivity analysis determines the change in the outcome or the reaction in the
environment given a change in a variable affecting such environment. Choice-letter
“c” is correct, because sensitivity analysis determines how the optimal solution will
react to changes in the parameters.
Choice-letter “a” is incorrect because the accuracy of the parameters are
assumed to have been tested in the sensitivity analysis. Choice-letter “b” is incorrect
because developing technological matrix may be related to linear programming and
not to the sensitivity analysis in linear programming. Choice-letter “d’ is incorrect
because sensitivity analysis is not used in developing the objective function used in
linear programming.

80. The graphic method as a means for solving linear programming problems
A. Can be used given more than two restrictions (constrains).
B. Is limited to situations having two restrictions (constraints).
C. Is limited to situations with one restriction (constraints).
D. Cannot be used with any restrictions (constraints).

80. B
? The item depicted on the linear programming graph.
 Choice-letter “b” is correct. Using the graphical method of solving linear programming
problems, the equations are depicted on the graph and the corner point in the shaded
area serve as possible optimal solutions. The line of best fit is not depicted on the
graph because linear programming model does not find the best representation
among the observations plotted on the graph but rather shows the linearity of an
equation depicted on the graph.
Choice-letter “a”, “c”, and “d” are incorrect because they do not fit with the correct
representation on the linear programming graph.
Chapter 22  Quantitative techniques in business 188

81. The Lacquers “R” Us Company manufactures lacquered jewelry boxes (J) and
lacquered mirrors (M). these data are presented to you as financial consultant:
1. The J’s need 3 kilos of material and 3 hours of labor; the M’s need 1.5
kilos of material and 1 hour of labor.
2. Materials costs P4.00 a kilo and labor costs P20.00 an hour.
3. P20,000 in fixed factory overhead is expected for the month’s population.
4. To be available is 400 kilos of material and 240 hours of labor.
5. Contribution margin ration for J is 40% and 20% for M.
The objective function for Lacquers “R” Us Company will be:
A. Maximize: Contribution Margin = P.4J + P.2M.
B. Maximize: Contribution Margin = P48J+P6.5M-P20,000.
C. Maximize: Contribution Margin = P48J+P6.5M.
D. Maximize: Contribution Margin = P28.8J+P5.2M.
(rpcpa)

81. C
? The objective function for lacquers “R” Us Company.
 The short-term objective function is always to optimize profit by either maximizing
revenue or maximizing costs. The objective function to optimize profit for products J
and M is represented by either contribution margin which is determined for each
product as follows:
Product J Product M
Materials cost per unit
(P4 x 3 kgs.) P12.00
(P4 x 5 kgs.) P6.00

Direct labor cost per unit


(P20 x 3 hrs.) 60.00
(P20 x 1 hr.) _______ 20.00
Unit variable costs P72.00 P26.00
Variable cost ratio 60% 40%
Unit contribution margin
(P72/60% x 40%) P48.00
(P26/40% x 60%) P6.50

To get the unit contribution margin (UCM), the unit variable costs must be divided
by the VCRatio to get the amount of sales and then multiplied by the CMRatio.
If J represents product J and M represents product M, then the objectives
function is:
Maximize: CM = 48J + 6.50M

Network models (Project scheduling)


82. The network below describesB
the interrelationships
1 of several activities necessary to
complete a project. The arrows represent the D activities. The numbers between the
arrows indicate the number
2 of months to complete each2 activity.

3 F End

A
3
1
C 2
E
Chapter 22  Quantitative techniques in business 189

Start

The shortest time to complete the project is


A. 5 months. C. 8 months.
B. 6 months. D. 14 months.

82. C
? The shortest time to complete the project.
 The shortest time to complete the project is the critical path. It is the longest path
among all the paths to be undertaken to complete a project. The longest path is the
critical path because a delay in the critical path would mean a delay in the entire
project while a delay in other paths may not mean a delay in the entire project. In
project management, time reduction and cost reduction are two of the most important
items to manage.
On the given network, the possible paths to complete the project as well as their
corresponding time to complete are as follows:
Paths Time
A-B-D-F 2+1+2 = 5
A-C-D-F 3+3+2 = 8 (longest path = critical path)
A-C-E-F 3+2+1 = 6

The critical path is path A-C-D-F having a completion time of 8 months, choice-
letter “c” is correct. Choice-letters “a” and “b” are incorrect because they are shorter
paths to reach the last activity “F” and therefore are not the most critical path in the
overall completion of the project. Choice-letter “d” is incorrect because there is no
path with a completion time of 14 months.

83. California Building Corporation uses the critical path method to monitor construction
jobs. The company is currently 2 weeks behind schedule on Job #181, which is
subject to a P10,500-per-week completion penalty. Path A-B-C-F-G-H-I has a
normal completion time of 20 weeks, and critical path A-D-E-F-G-H-I has a normal
completion time of 22 weeks. The following activities ca be crashed:
Cost to Crash Cost to Crash
Activities 1 Week 2 Weeks
BC P 8,000 P 15,000
DE 10,000 19,600
EF 8,800 19,500
California Building desires to reduce the normal completion time of Job #181 and, at
the same time, report the highest possible income for the year. California Building
should crash
A. Activity BC 1 week and activity EF 1 week.
Chapter 22  Quantitative techniques in business 190

B. Activity DE 1 week and activity BC 1 week.


C. Activity EF 2 weeks.
D. Activity DE 1 week and activity EF 1 week.

83. D
? The path(s) a company should crash to accelerate the project’s completion time by 2
weeks.
 If a firm wants to speed up the completion of a project or catch up a project’s
completion time, an activity(ies) that should be crashed (or accelerated).
As a rule, the activity(ies) to be crashed should be found on the critical path. The
critical path is A-D-E-F-G-H-I and the activities found on the critical path are DE and
EF. Activity BC does not belong to the critical path activities and therefore would not
be a priority for crashing. An analysis of cost effects in crashing activities DE and EF
are as follows:
Cost to crash Cost to crash in Total cost to
Activity in the first week the second week crash in 2 weeks
DE P10,000 P9,600 P19,600
EF 8,800 9,700 19,500
Based on the data, it would be more profitable for the California Building
Corporation to crash 1 week of activity EF in the first week and 1 week of activity DE
in the second week, with a total crash cost as follows:
Time to crash Cost to crash
EF 1 week P 8,800
DE 1 week 9,600
Total P18,400
As such, choice-letter “d” is correct. Choice-letters “a” and “b” are incorrect
because activity BC should not be crashed since it is not found on the critical path.
Choice-letter “c” is incorrect because crashing activity EF by 2 weeks would give a
total crash cost of P19,500 which is higher than that of crashing 1 week of EF and 1
week of DE for a total cost of P18,400.

84. When using PERT, the expected time for an activity when given an optimistic time
(a), a pessimistic time (b) and a most likely time (m), is calculated by which one of the
following formulas?
A. (b – a)  2. C. (a + 4m + b)  6.
B. (a + b)  2. D. (4abm)  6.

84. A
? The formula used in determining the expected time used in the PERT analysis.
 Choice-letter “c” is correct. The expected time is the weighted average time of
completing a particular activity. The expected time is based on the average of
optimistic time, most likely time and pessimistic time where the assigned points are 1,
4, and 1 for optimistic, most likely, and pessimistic time, respectively. This
assignment of values is based on the two-tail normal distribution curve. In this two-
tail normal distribution, the acceptance region is assigned 4 points and each tail is
assigned a point, thereby a total of 6 points. The formula to compute the expected
time is:
Chapter 22  Quantitative techniques in business 191

Expected time = (Optimistic time + 4(Most likely time) + Pessimistic time) / 6


If: a = optimistic time
m = most likely time
b = pessimistic time
Then: Expected time = (a + 4m + b) / 6

85. In a PERT network, the optimistic time for a particular activity is 9 weeks, and the
pessimistic time is 21 weeks. Which one of the following is the best estimate of the
standard deviation for the activity?
A. 2 C. 9
B. 6 D. 12

85. A
? The best estimate for the standard deviation of an activity in a PERT network.
 Standard deviation relates to the difference of actual observation from that of the
middle (or average) observation. In PERT analysis, the computation of expected time
is averaged by the constant factor “6”. Therefore, the standard deviation of the given
activity should be:
Standard deviation = (Optimistic time – Pessimistic time) / 6
= (21 – 9) / 6 = 12 / 6 = 2

Questions 86 and 87 are based on the following information: The PERT network
diagram and corresponding activity cost chart for a manufacturing project at
Networks, Inc. is presented below. The numbers in the diagram are the expected
times (in days) to perform each activity in the project.

C
5.0 6.5
1.0
4.5 7.5
A
B E
5.5 0.5 7.5

Normal Crash
Activity Cost Crash Time Cost
AB P3,000 3.50 days P4,000
AC 5,000 4.50 5,250
AD 4,000 4.00 4,750
BE 6,000 5.00 7,000
CE 8,000 5.00 9,200
DE 6,000 6.50 6,750
BC 2,500 0.50 3,500
BD 2,000 0.25 2,500
Chapter 22  Quantitative techniques in business 192

86. The expected time of the critical path is


A. 12.0 days. C. 11.5 days.
B. 13.0 days. D. 12.5 days.

86. B
? The expected time of the critical path.
 The expected time of the critical path is 13 days as follows:
Path Expected time
A-C-E 5.0 + 6.5 = 11.5 days
A-B-C-E 4.5 + 1.0 + 6.5 = 12.0
A-B-E 4.5 + 7.5 = 12.0
A-B-D-E 4.5 + 0.5 + 7.5 = 12.5
A-D-E 5.5 + 7.5 = 13.0 (critical path)

Choice-letter ”b” is correct because the critical path is the one that has the
longest expected time to complete.

87. To keep costs at a minimum and decrease the completion time by 1 ½ days,
Networks, Inc. should crash activity(ies)
A. AD and AB. C. AD.
B. DE. D. AB and CE.

87. A
? Te activity(ies) to be crashed to keep costs at a minimum and decrease the
completion time by 1.5 days.
 The first activity to be crashed should be on the critical path A-D-E. Activity AD could
be crashed by 1.5 days (5.5 – 4.0) while activity DE could be crashed by 1.0 day only
(7.5 – 6.5). The choice should therefore be to crash AD since it meets the 1.5 days
reduction in completion time. Still, however, the other paths in the network should be
crashed because the desired time to complete is 11.5 days (13.0 – 1.5). The needed
time to crash the other paths is as follows:
Desired time
Path Expected time to complete Time to crash
A-C-E 11.5 11.5 0
A-B-C-E 12.0 11.5 0.5
A-B-E 12.0 11.5 0.5
A-B-D-E 12.5 11.5 1.0
A-D-E 13.0 11.5 1.5
Paths ABCE, ABE, and ABDE are likewise to be crashed. The common activity
among the paths is activity AB. Crashing this activity would reduce the completion
time by 1 day (i.e., 4.5 – 3.5). Therefore, the paths to be crashed are:
AD 1.5 days
AB 1.0 day
Choice-letter “b” is incorrect because crashing path DE would only mean a day of
crashing and does not meet the 1.5 days requirement. Choice-letter “c” is incorrect
because crashing only path AD would not accelerate the completion of the entire
project by 1.5 days because the other paths are not yet crashed. Choice-letter “d” is
Chapter 22  Quantitative techniques in business 193

incorrect because crashing path CE would be more costly since it still needs to crash
path AB.

88. The Gantt chart below shows that the project is


Now
Time (weeks)
Activity 2 4 6 8 10 12 14 16

A
B
C
D

A. Complete. C. On schedule.
B. Ahead of schedule. D. Behind schedule.

88. B
? The information that could be derived from the Gantt chart with respect to a project
that is on its 11th week of working.
 The shaded activities signify completion of the activity. Reading the chart, the project
is now in its 9th week. Activity A is already completed as scheduled, activity B has
already an equivalent completion of up to the 13 th, and activity C is already completed
which should have been completed by the 15th week. These data mean that the
project is ahead of schedule. Hence, choice-letter “b” is the correct answer.
Choice-letter “a” is incorrect because the project is not yet completed. Choice-
letters “c” and “d” are incorrect because the project is not on schedule or behind
schedule, but is ahead of schedule.

89. A bank is designing on on-the-job training program for its branch managers. The
bank would like to design the program so that participants can complete it as quickly
as possible. The training program requires that certain activities be completed before
others. For example, a participant cannot make credit loan decisions without first
having obtained experience in the loan department. An appropriate scheduling
technique for this training program is
A. PERT-CPM. C. Queuing theory.
B. Linear programming. D. Sensitivity analysis.

89. A
? An appropriate training technique for the training program.
 Choice-letter “a” is correct. The training program requires that certain activities be
completed before others and to be completed as quickly as possible. This
necessitates proper scheduling of activities which needs the application of PERT-
CPM.
Choice-letter “b” is incorrect because linear programming deals with the
optimization of scarce resources and not on the scheduling of activities. Choice-letter
“c” is incorrect because queuing theory deals with the issue of maximizing the benefit
Chapter 22  Quantitative techniques in business 194

of a service facility in relation to the cost of waiting a service. Choice-letter “d” is


incorrect because sensitivity analysis determines the possible outcome or effects of a
change in a variable affecting such outcome.

90. PERT and the critical path method (CPM) are used for
A. Determining the optimal product mix.
B. Project planning and control.
C. Determining product costs.
D. Determining the number of servers needed in a fast food restaurant.

90. B
? The use of PERT and CPM.
 Choice-letter “b” is correct, PERT and CPM are used for project planning and control
to complete the project as fast as possible at the least cost involved. Choice-letter
“a” is incorrect because determining the optimal product mix is a concern of linear
programming. Choice-letter “c” is incorrect because determining product costs is a
concern of cost accounting. Choice-letter “d” is incorrect because determining the
number of servers needed in a fast food restaurant is an issue involving the
application of queuing models.

91. The primary difference between PERT and CPM is that


A. CPM uses probabilities on the activity times and PERT does not.
B. PERT considers activity costs and CPM does not.
C. PERT can assign probabilities to activity times and CPM does not.
D. CPM considers activity costs and PERT does not.

91. D
? The primary difference between PERT and CPM.
 Choice-letter “d” is correct, CPM considers activity costs while PERT does not. It
makes choice-letter “b” an incorrect choice. Choice-letters “a” and “c are incorrect
because both CPM and PERT use and assign probabilities to activity time.
A network model that considers only activity costs is PERT. Once, a network
model considers activity costs it becomes a CPM.

92. In a PERT (Program Evaluation Review Technique) or CPM (Critical Path Method)
network, the critical path is the
A. Least costly path through the network.
B. Most costly path through the network.
C. Shortest path through the network with the least amount of slack.
D. Longest path through the network with the least amount of slack.

92. D
? The description of the critical path in the PERT and CPM.
 A path of activities becomes critical in the project scheduling when it has the longest
time in the network. It is critical because it does not allow delays or slack in the
project completion. The other paths may be delayed and still would not cause a
delay in the entire project. But once the critical path delays, the entire project is
delayed. Choice-letter “d’ is correct.
Chapter 22  Quantitative techniques in business 195

Choice-letters “a” and “b” are incorrect because the primary consideration in the
critical path is the time and not the cost. Choice-letter “c” is incorrect because the
shortest path gives the most of the possible allowance for delays without affecting the
completion time of the entire project and still would not affect the project profitability.

93. When using the PERT method for network analysis, the critical path through the
network is
A. The longest path through the network.
B. The shortest path through the network.
C. The path with most slack.
D. The least cost path. (rpcpa)

93. A
? The critical path in the PERT analysis.
 The Program Evaluation Review Technique (PERT) model is used to aid managers
in controlling large-scale, complex projects. It determines the probabilistic time
estimates and identifies the critical path. Critical path is the longest path in time
through the network. It is critical because if an activity on the critical path takes longer
than expected, the entire project will be delayed.

94. Which of the following statements is the least to the Project Evaluation Review
Technique (PERT).
A. It is a system, which uses network analysis and critical path method.
B. It is more useful for analyzing the interrelationships of time and activities to
discover potential bottlenecks.
C. It involves measuring progress in relation to schedule, evaluating changes to
schedule, forecasting future progress and predicting and controlling costs.
D. Time, is a primary consideration and this technique is particularly suited for
problems, which involve the combination of resources that maximize profits or
minimize costs. (rpcpa)

94. D
? The statement that is the least pertinent with respect to Program Evaluation Review
Technique (PERT).
 PERT is used in complex projects where there is a presence of numerous activities to
be undertaken before completion. Some of the activities can be done simultaneously
while other are done in sequence.
Statements “a”, “b”, and “c” are descriptive of PERT. It uses network analysis.
Its usefulness is enhanced by the critical path method. It shows each activity as line
between events. A sequence of line shows interrelationships among activities which
are the tasks to be accomplished that consume resources, including time, and have a
duration over time. PERT identifies the potential bottlenecks in completing a project.
It involves measuring progress in relation to schedule, evaluating changes to
schedule , forecasting future progress, and predicting and controlling costs.
Statement letter “d” is the correct answer because it refers to linear programming
where time (i. e., labor hours, machine hours) is considered as a scarce resource.

95. A Gantt chart


Chapter 22  Quantitative techniques in business 196

A. Shows a critical path of a project.


B. Is used for determining an optimal product mix.
C. Shows only the activities along the critical path of a network.
D. Does not necessarily shoe the critical path through a network.

95. D
? A description about a Gantt chart.
 A Gantt chart shows the activities to be undertaken with their estimated completion
time. In this chart, you could discern the activities that could be done simultaneously
and those to be done after an activity is completed. It also provides the project
manager the estimated time to complete the project. However, the Gantt chart does
not show the critical path. Choice-letter “d” is correct.
Choice-letter ‘a” is incorrect. Choice-letter “b” is incorrect because a Gantt chart
does not determine the optimal product mix which is determinable using the linear
programming model. Choice-letter “c’ is incorrect because the Gantt chart shows all
the activities in a project and not only those in the critical path.

96. In the Program Evaluation Review Technique(PERT), slack is the


A. Uncertainty associated with time estimates.
B. Path that has the largest amount of time associated with it.
C. Excess time available in the completion of the project after crashing the critical
path.
D. Number of days an activity can be delayed without forcing a delay for the entire
project.

96. D
? A description of slack in PERT.
 A slack is the difference between a completion time of a given activity path with that
of the completion time of the shortest activity path. Slack represents the possible
reduction in time to complete the activity path that may bring savings in the project.
Looking at it in a different window, slack is a number of time period (i.e., days) an
activity can be delayed without forcing a delay for the entire project. Choice-letter “d’
is correct.
Choice-letter “a” is incorrect because the uncertainty associated with time
estimates is a matter of risk analysis (or uncertainty or probability analysis) and not
the determination of slack time. Choice-letter “b’ is incorrect because it refers to the
critical path. Choice-letter “c” is incorrect because slack refers to the possible time to
accelerate the completion of the project and the excess time available in the
completion of the project after crashing the critical path.

97. When making a cost/time trade-off in PERT analysis, the first activity that should be
crashed is the activity
A. On the critical path with the lowest unit crash cost.
B. On the critical path with the maximum possible time reduction.
C. With the lowest unit crash cost.
D. With the largest amount of slack.

97. A
Chapter 22  Quantitative techniques in business 197

? The first activity that should be crashed when making a cost/time trade-off in PERT
analysis.
 Choice-letter “a” is correct because the first activity to be crashed should always be
on the critical path with the lowest crash cost. Choice-letter “b” is incorrect because
crashing with the maximum possible time reduction may be the most economical
option of accelerating the completion time of the project. Choice-letter “c” is incorrect
because the lowest crash cost is not the primary consideration but the time reduction
on the critical path. Choice-letter “d” is incorrect because crashing the path with the
largest amount of slack speaks only of time reduction without considering the costs of
crashing.

98. Of these statements, which is the least pertinent to the concept of “slack” in relation
to the Project Evaluation and Review Technique (PERT)?
A. The less the amount of slack time, the more critical an activity or path
B. Slack time information is useful for planning and continuous monitoring
C. It is computed by subtracting the earliest expected time from the earliest
allowable time
D. If not exceeded, non-critical activities can be delayed without delaying the
project’s completion time (rpcpa)

98. A
? A true statement abut “slack” in relation to the project Evaluation and Review
Technique (PERT)?
 PERT is a technique in project monitoring activities where the critical path is identified
to avoid delay in completing a project. Critical path is the longest path of completing
the project.
Choice-letters “b”, “c”, and “d” are statements that are pertinent with regard to
slack. Slack time is the difference between the critical path and the earliest possible
time to complete the project. It represents the unused resources that can be diverted
to the critical path. If not exceeded, these unused resources (or noncritical activities)
may be delayed without delaying the project’s completion time. And slack time
information is used in the planning and continuous monitoring of activities in the
completion of a project.
Choice- letter “a” is correct because a lesser slack time means that the project is
less critical and not more critical. Choice-letter “a” is the least pertinent to the
concept of slack.

99. Which of the following statements best describes a difference between basic PERT
and the Critical Path Method (CPM) of network analysis?
A. PERT uses probability distributions on the activity times while CPM uses point
estimates for the activity times
B. PERT does not allow for slack times o n the activities while CPM does
C. PERT does not consider activity cost while CPM does
D. PERT determines the least-cost path through a network while CPM determines
the least-time path through a network. (rpcpa)

99. C
Chapter 22  Quantitative techniques in business 198

? A statement best describes the difference between PERT (Program Evaluation


Review Technique) and CPM (Critical Path Method).
 Both PERT and CPM are network analysis techniques. PERT may be considered as
the initial phase in network analysis before moving to CPM.
Choice-letter “a” is incorrect because both PERT and CPM use probability
distributions. Choice-letter “b” is incorrect because both PERT and CPM consider
and aims to eliminate slack time on the activities Choice-letter “d” is incorrect
because PERT does not determine the least-cost but only the least-time path through
a network.

Queuing Theory
100.A company has several departments that conduct technical studies and prepare
reports for clients. Recently, there have been long delays in having these reports
copied at the company’s centralized copy center because of the dramatic increase in
business. Management is considering decentralizing copy services to reduce the
turnaround and provide clients with timely reports. An appropriate technique for
minimizing turnaround time and the cost of providing copy service is
A. Queuing theory. C. Regression analysis.
B. Linear programming. D. Game theory.

100.A
? An appropriate technique for minimizing turnaround time and the cost of providing
copy service.
 This case comes into a situation where the service facility simply is no longer
adequate to accommodate the need for such services. One way to address the
situation is to increase the number of service facility vis a vis the reduction in the cost
of waiting. This situation calls for the application of the queuing model; choice-letter
“a” is correct.
Choice-letter “b” is incorrect because linear programming is applied for the
optimization of limited resource. Choice-letter “c” is incorrect because regression
analysis estimates the value of the dependent variable in relation to the behavior of
the independent variable. Choice-letter “d” is incorrect because game theory is used
in expecting the reaction of a competitor in the market place given the course of
action to be undertaken by the firm.

101.Queuing models are concerned with balancing the cost of waiting in the queue with
the
A. Cost of providing service.
B. Number of customers in the queue.
C. Average waiting time in the queue.
D. Usage rate for the service being rendered.

101.A
? The concern of the queuing models.
 Choice-letter “a” is correct. Queuing theory (or waiting line analysis) is concerned
with balancing the cost of providing the service facility with that of the cost of waiting
if the service is not provided. To be feasible, the savings derived in providing the
Chapter 22  Quantitative techniques in business 199

service arising from the reduction of the cost of waiting should be higher than the cost
of providing the service facility.
Choice-letters “b” and “c” are incorrect because the number of customers on the
queue (or line) or the average time in the queue does not necessarily represent the
cost needed to recover the savings from reducing the waiting line. Choice-letter “d” is
incorrect because the usage rate for the service being rendered, just like choice-
letters “b” and “c”, is a factor in determining the cost of waiting rather than a factor in
determining the cost of providing the service facility.

102.The operating condition that cannot be identified by using a queuing model is the
A. Average percentage of time that a service facility is idle.
B. Probability of a specified number of units in the queue.
C. Actual amount of time each unit spends in the queue.
D. Average number of units in the system and the mean length of the queue.

102.C
? The operating condition that cannot be identified by using a queuing model.
 The queuing model balances the cost of providing the service with the reduction in
cost of waiting in the queue. The cost of providing the service facility shall be
determined by the cost of equipment and personnel as well as the opportunity cost of
using the service in other undertaking while it is idle. The reduction in the cost of
waiting is determined by the average number of units in the queue, the probability of
a specified number of units in the queue, the cost spent in maintaining the units in the
queue, and the possible number of units that could be reduced in the queue if the
service facility is provided. Choice-letters “a”, “b”, and “d” are important in
determining the operating condition in the queue environment. Choice-letter “d”,
actual amount of time each unit spends in the queue, is not an important variable in
the operating condition of the queue environment, hence the correct answer.

103.The drive-through service at a fast food restaurant consists of driving up to place an


order, advancing to a window to pay for the order, and then advancing to another
window to receive the items ordered. This type of waiting line system is
A. Single channel, single phase. C. Multiple channel, single phase.
B. Single channel, multiple phase. D. Multiple channel, multiple phase.

103.B
? A type of waiting line described in the situation.
 Choice-letter “b” is correct. The waiting line is a single-channel, multiple-phase
system. Choice-letter “a” is incorrect because the line has multiple phases. Choice-
letters “c” and “d” are incorrect because the line has only a single channel.

104.A post office serves customers in a single line at one service window. During peak
periods, the rate of arrivals has a Poisson distribution with an average of 100
customers per hour and service times that are exponentially distributed with an
average of 60 seconds per customer. From this, one can conclude that the
A. Queue will expand to infinity.
B. Server will be idle one-sixth of the time.
C. Average rate is 100 customers per hour.
Chapter 22  Quantitative techniques in business 200

D. Average customer waiting time is 2.5 minutes.

104.A
? A conclusion that could be derived if the average number of customer arrival is 100
per hour and the service time is 60 seconds per customer.
 Since the average arrival of customers is faster than the average service time, the
queue would likely expand to infinity; hence, choice-letter “a” is correct. Choice-letter
“b’ is incorrect because the server would not be idle and would be used all the time.
Choice-letter “c” is incorrect because the average rate of customers served would
only be 60 in an hour. Choice-letter “d” is incorrect because the average customer’s
waiting time will increase as the number of customers in the queue also increases.

Quality Tools
105.Valor Company produces a component of a machine. The target value for a key
dimension of the component is 100 millimeters (mm). The quality loss per unit if the
key dimension is measured at precisely the upper or lower specification limit (100mm
+ 1 mm) is estimated to be P10.00. The following are the measures of a sample of
four units:
Unit Measurement
1 99
2 101
3 99.5
4 100.5
Valor Company produces 1,000 of the components. Based on the sample, the
estimated quality loss is
A. P 625 C. P10,000
B. P6,250 D. P10,000

105.B
? The estimated amount of quality loss.
 The estimated amount of the quality loss shall be the estimated loss per unit
multiplied by the number of units produced. The estimated loss per unit shall be
determined based on the analysis of the samples. There are four (4) samples
observed and the average measurement is 100 mm. The quality loss shall be based
on the variance computed as follows:
Units X T (X – T)² k k(X-T) ²
1 99 100 1.00 P10 P10.00
2 101 100 1.00 10 10.00
3 99.5 100 0.25 10 2.50
4 100.5 100 0.25 10 2.50
P25.00
whrre: X = actual measurement
T = target measurement
k = cost of quality loss per unit

The average loss per unit is P6.25 (i.e., P25/4), and the estimated total loss is
P6,250 (i.e., P6.25 x 1,000 units).
Chapter 22  Quantitative techniques in business 201

Questions 106 and 107 are based on the following information. An organization has
collected data on the complaints made by personal computer users and has
categorized the complaints, as follows:

40 n = 93

30

20

10

0
Lack
ofHardwareDisketteAdvanced/UserProblemsProblemsNewKnowledgeSoftw
areUsage
Types of Complaints

106.Using the information collected, the organization should focus on


A. The total number of personal computer complaints that occurred.
B. The number of computer complaints associated with diskette problems and new
software usage.
C. The number of computer complaints associated with the lack of user knowledge
and hardware problems.
D. The cost to alleviate all computer complaints.

106.C
? The variable in which the organization should focus.
 The variables where the organization should focus is to be based on the data
observed on the presented graph. The graph tells that the most number of nominal
complaints are the lack of user’s knowledge and hardware problems. Choice-letter
“c” is correct.
Choice-letter “a” is incorrect because allocating the limited managerial time to
address problems should be focused on the most recurring complaints and not on all
complaints including the insignificant ones. Choice-letter “b” is incorrect because the
number of diskette problems and new software usage are significantly less in the
number of complaints and should be subordinated to the need of attending to the
most numbered complaints. Choice-letter ‘d” is incorrect because the organization
should not focus on the cost of alleviating computer complaints but rather in
minimizing complaints.

107.The chart displays


A. The arithmetic mean of each computer complaint.
Chapter 22  Quantitative techniques in business 202

B. The relative frequency of each computer complaint.


C. The median of each computer complaint.
D. The absolute frequency of each computer complaint.

107.D
? The one displayed on the chart.
 The chart displays the absolute frequency of each computer complaint, choice-letter
‘d” is correct. Choice-letters “a” and “c” are incorrect because the chart displays the
absolute or nominal frequency of the observation and not the arithmetic mean or the
median of each computer complaint. Choice-letter “b” is incorrect because relative
frequency refers to the percentage distribution of the observation on the chart.

 done 

You might also like